[obm-l] Re: [obm-l] Re: [obm-l] Re: [obm-l] Dúvida

2019-12-05 Por tôpico Bernardo Freitas Paulo da Costa
Oi Pedro e Pedro, e demais colegas da OBM-L

Eu também nunca lera a definição de elipses através da razão entre as
distâncias.  Achei interessante, porque talvez permita "interpolar"
entre elipses, parábolas e hipérboles.  Mas até hoje, todas as
definições que eu vira de elipses (inclusive a da soma das distâncias)
incluíram círculos.  Mas, como talvez tenha desejado indicar o Pedro
Fonini, o importante é *para que serve a definição*.  No caso das
elipses, é muitas vezes importante incluir os círculos com elas, por
exemplo para o teorema de álgebra linear que ele citou.  Talvez o caso
da definição por razão das distâncias indique um outro caminho, mas aí
minha impressão é que o caso seria que a reta diretriz está no
infinito para os círculos (o que, mais uma vez, reforça a unidade das
cônicas no plano projetivo).

Acho que o mais comum, *hoje em dia*, é definir elipses de forma a
incluir os círculos.  Talvez isto invalide a definição que você deu
via razões, que passa a ser um teorema apenas para as elipses com dois
focos distintos.  Mas, por outro lado, permite generalizar de forma
mais natural outros teoremas para os quais a inclusão dos círculos
como elipses simplifique o enunciado.  Talvez você prefira a definição
por razões, mas voltando às origens das cônicas, onde a "classe" é
determinada pela posição relativa do plano secante com relação ao cone
gerador, acredito que a inclusão dos círculos junto com as elipses
seja totalmente razoável.

Cônicas suaves me parecem um assunto avançado; esta terminologia mesmo
já faz pensar em funções (infinitamente) diferenciáveis, etc, típicas
o ensino superior.  Do ponto de vista da geometria algébrica, uma
cõnica é definida como zeros de um polinômio P(x,y) de grau dois,
então esta "definição" não pode servir para separar quem seja suave e
quem não seja.  Se há uma diferença entre um círculo e duas retas que
se intersectam, não é pela regularidade da função que os define
implicitamente: num caso é x^2 + y^2 = 1, no outro, xy = 0.  O que
acaba servindo é a definição de "variedade suave".  E daí eu estaria
puxando mais ainda para temas universitários...

On Wed, Dec 4, 2019 at 9:33 PM Pedro José  wrote:
>
> Boa noite!
> As retas são cônicas degeneradas. Mas são cônicas.
> Definição de cônica :   Dada duas retas g,l concorrentes (cuja interseção é 
> {V} no |R3 que não sejam perpendiculares e um plano Pi. A interseção desse 
> plano com o cone K, reto de vértice V e eixo l , obtido pela rotação da reta 
> g ao redor do ponto V é uma cônica. Podemos ter uma reta, duas retas ou um 
> ponto como cônicas degeneradas.
> Você poderia até ter mencionado o conjunto vazio que não é uma cônica. 
> x^2+y^2=-1.
> Mas na verdade, eu não me expressei com rigor, o que queria dizer é que se 
> escrevermos a função quadrática F(x,y)= 0, que represente a cônica 
> (degenerada ou não) F(x,y) é suave? Ou as cônicas suaves devem ser não 
> degeneradas apenas?
> Outrossim, discordo do seu argumento "...geralmente é mais útil que as 
> definições dos objetos importantes não excluam os casos particulares.."
> Geralmente não é o balizador e sim a definição.
> 1 não é primo. Pois define-se que um primo deve ter dois divisores positivos 
> e 1 só possui um. Poderia argumentar, na sua linha, os dois divisores 
> coincidentes (os que afirmam é divisível por si e pela unidade)
> O quadrado por definição está claro que é retângulo.
> A definição da elipse é de que a soma das distâncias a dois pontos fixos (e 
> não um) é constante. Aí tem a forçação de se considerar dois como um só. Não 
> existe dois pontos coincidentes. Se são dois são distintos. Podemos 
> representar algo de várias maneiras mas se são iguais é só um, representado 
> de várias maneiras. Qual o cardinal do conjunto de focos de uma elipse, no 
> caso de você aceitar a elipse com um único foco?
> Como é a prova que só existe um vazio. Por hipótese há mais de um vazio, 
> vazio1 e vazio2 e no fim chega-se a conclusão que vazio1 = vazio2 e portanto 
> absurdo.Ora, podemos ter vazios coincidentes.
> Amigo, você afirma: "Nunca vi ninguém definir elipse de uma forma que exclua 
> os círculos."
> Você nem se deu ao trabalho de ler a minha nota, antes de comentar, ou então 
> me corrija se o círculo atende à:
> Lugar geométrico do plano em que a razão entre a distância de um ponto ao 
> foco direito e a distância entre esse ponto e uma reta (diretriz direita) é 
> constante e menor que 1 e igual a excentricidade da cônica.
> Como a excentricidade da circunferência é zero, teríamos que ter um ponto 
> fixo em que a distância de cada ponto da circunferência até esse ponto fosse 
> zero. E se na definição tem foco direito está implícito que há um esquerdo. 
> Vale a definição para foco esquerdo. Só atenderia se considerarmos o ponto 
> como uma circunferência de raio zero. E só para esse caso e ainda aceitarmos 
> que quando há só um foco ele tanto é direito quanto esquerdo. Grato pelos 
> comentários. Mas as dúvida persistem.
>
> Saudações

[obm-l] Re: [obm-l] Re: [obm-l] Re: [obm-l] Dúvida basica equação polar

2019-09-02 Por tôpico Ralph Teixeira
Pois bem, se voce parametrizar com relacao ao centro, teria
x(teta)=1+cos(teta) e y(teta)=sin(teta). Se fosse assim, teria que ser
0 wrote:

> Caro Ralf, obrigado pela resposta.Para mim ficou confuso pq pensei que a
> parametrização do círculo se daria colocando como referencia o novo centro
> do mesmo. Quando penso em circulos diferentes , por exemplo residindo em
> apenas um quadrante tenho dificuldade de imaginar varrendo todos os pontos
> . Vou refletir sobre esses casos pois parecem ser obtidos como vc disse de
> fato.
>
> Att.Gabriel
>
> Em Seg, 2 de set de 2019 18:04, Ralph Teixeira 
> escreveu:
>
>> Bom, vale a pena fazer uma figura primeiro... Fez? Note como este circulo
>> estah nos primeiro e quarto quadrantes apenas.
>>
>> Entao suponho que voce fez as contas e descobriu que r=2cos(teta). No
>> quarto quadrante vale -pi/2> onde pi/2> terceiro quadrante). Por isso que -pi/2>
>> Para ser mas exato, o que aconteceria na equacao r=2cos(teta) para teta
>> entre pi/2 e 3pi/2... Tipo, experimente pensar em teta=pi para fazer um
>> exemplo. Jogando na equacao, ficaria r=2cos(pi)=-2?!?
>>
>> Aqui ha duas opcoes:
>> a) Alguns livros vao insistir que r>=0 sempre. Neste caso, fica claro que
>> pi/2> cosseno fica negativo.
>> b) Alguns outros livros sao mais "liberais" e permitem r<0 -- a
>> interpretacao seria que quando r eh negativo voce anda na reta que forma
>> angulo teta com o eixo x NO SENTIDO OPOSTO. Por exemplo, r=-2 e teta=pi eh
>> de fato o ponto (2,0) (voce anda na direcao NEGATIVA do eixo x, mas voce
>> anda -2, entao acaba andando para a DIREITA duas unidades). Neste caso,
>> colocar teta=pi dah um ponto no circulo sim senhor! Mas, mesmo assim, eu
>> usaria apenas -pi/2> ponto (2,0) JAH APARECEU com teta=0, e nao vejo porque conta-lo duas vezes
>> (e, dependendo da aplicacao, voce NAO QUER contar cada ponto duas vezes).
>>
>> Abraco, Ralph.
>>
>> On Mon, Sep 2, 2019 at 4:55 PM Gabriel Lopes  wrote:
>>
>>> Boa tarde, tenho uma duvida básica da representação em equação polar do
>>> círculo  (x-1)^2 +y^2= 1.
>>>
>>> Pq os intervalo de teta é de -pi/2 a pi/2 e nao de 0 a 2pi?
>>>
>>> --
>>> Esta mensagem foi verificada pelo sistema de antivírus e
>>> acredita-se estar livre de perigo.
>>
>>
>> --
>> Esta mensagem foi verificada pelo sistema de antivírus e
>> acredita-se estar livre de perigo.
>
>
> --
> Esta mensagem foi verificada pelo sistema de antivírus e
> acredita-se estar livre de perigo.

-- 
Esta mensagem foi verificada pelo sistema de antiv�rus e
 acredita-se estar livre de perigo.



[obm-l] Re: [obm-l] Re: [obm-l] Re: [obm-l] Dúvida

2019-05-24 Por tôpico Anderson Torres
Em dom, 19 de mai de 2019 às 13:24, Pedro José 
escreveu:

> Bom dia!
> Anderson,
> obrigado. Porém faltou-me saber se os entendimentos anteriores estão
> corretos.
>

O texto não tinha nenhum glossário para ajudar, ou uma referência do
gênero? Alguns bons livros de Teoria dos Números, em especial os grossões
como o do Hua Loo-Keng, costumam dedicar a primeira folha a convenções.

Eu até suspeito que esses teus entendimentos estejam corretos - dado que
são bem parecidos com outros já consagrados pelo uso - mas não bato o
martelo porque não tenho o contexto.



> Grato,
> PJMS
>
> Em sáb, 18 de mai de 2019 13:27, Anderson Torres <
> torres.anderson...@gmail.com escreveu:
>
>>
>>
>> Em sex, 17 de mai de 2019 às 10:49, Pedro José 
>> escreveu:
>>
>>> Bom dia!
>>>
>>> Tenho uma dúvida sobre os simbolismos, que aparecem recorrentemente, em
>>> artigos sobre teoria dos números, mas que não encontro a definição :
>>> Z[i]/(α) - Entendi como o conjunto das classes de equivalências mod α
>>> em Z{i}
>>> Z[i]/(α)* - Entendi como as classes de equivalência mod α em Z[i], que
>>> são inversíveis.
>>>
>>> Tentando compreender uma demonstração de que todos os números que não
>>> podem ser escritos da forma 4^k(8m+7) com k,m>=0, aceitam ser escritos como
>>> a soma de três parcelas, todas quadrados, me deparei com  [image:
>>> image.png] . O que significa?
>>>
>>
>> Algo como números da forma a+b*sqrt(m) onde a e b são racionais.
>>
>>
>>
>>>
>>> Os outros entendimentos estão corretos?
>>>
>>> Saudações,
>>> PJMS
>>>
>>>
>>> --
>>> Esta mensagem foi verificada pelo sistema de antivírus e
>>> acredita-se estar livre de perigo.
>>
>>
>> --
>> Esta mensagem foi verificada pelo sistema de antivírus e
>> acredita-se estar livre de perigo.
>
>
> --
> Esta mensagem foi verificada pelo sistema de antivírus e
> acredita-se estar livre de perigo.

-- 
Esta mensagem foi verificada pelo sistema de antiv�rus e
 acredita-se estar livre de perigo.



[obm-l] Re: [obm-l] Re: [obm-l] Re: [obm-l] Re: [obm-l] Dúvida conceitual (equações)

2018-10-15 Por tôpico Bernardo Freitas Paulo da Costa
On Mon, Oct 15, 2018 at 8:07 AM Claudio Buffara
 wrote:
>
> Derivando e igualando a zero o lado esquerdo da sua equação, ficamos com:
> -2*cos(x)*sen(x) + sen(x) = 0 ==>
> sen(x) = 0  ou  cos(x) = 1/2 ==>
> x = 0 ou x = pi ou x = 2pi
> ou x = pi/3 ou x = 5pi/3.
>
> Assim, uma definição que me parece adequado para equações em geral (e não 
> necessariamente polinomiais) da forma f(x) = 0 é que uma raiz de 
> multiplicidade n é raiz de f, f’, ... , f^(n-1) mas não é raiz de f^(n).
>
> Naturalmente, se f não tiver todas as derivadas, precisaremos achar uma 
> definição diferente. Mas talvez, neste caso, nem faça sentido falar em 
> multiplicidade de uma raiz.

Essa definição funciona relativamente bem se f é analítica, porque o
comportamento local é determinado por inteiros.  Se f for apenas
diferenciável, talvez seja complicado dizer algo, como o exemplo
clássico de exp(-1/x^2).  A raiz tem multiplicidade infinita?

Enfim, existem, como você falou, boas razões para incorporar
multiplicidade (por exemplo estabilidade numérica), mas isso em geral
só faz sentido no mundo analítico, onde a noção de "grau" é dada pelas
derivadas.  Acho que mesmo no mundo C-infinito já pode haver
problemas, mas não sou especialista (nessas :D) patologias.  A questão
original, incluindo multiplicidades, pode ser resolvida simplesmente
usando as relações de Girard, que dependem de forma simples da
equação.

Vou tentar dar um exemplo que ilustra meu ponto de vista:  qual o
produto das raízes da equação x^2 - 4x + c?  "Qualquer um" dirá "c".
Mas, naturalmente, se c = 4, a única solução é x=2, e portanto (sem
usar multiplicidades) este produto seria apenas 2.  E daí a fórmula
fica muito mais complicada, com um caso especial, e descontínua.  A
grande sacada do Girard foi, justamente, propor incorporar as
multiplicidades, para simplificar as fórmulas (além, é claro, de
incluir também as soluções negativas, antes consideradas como
"absurdas" - este foi, provavelmente, o maior motivo de as pessoas
considerarem raízes negativas como algo que fazia sentido, e portanto
os números negativos também).  Mas isso não quer dizer que a equação
x^2 - 4x + 4 tenha duas soluções.  É apenas uma forma mais conveniente
de interpretar as raízes quando se pensam nas relações de Girard (e
várias outras fórmulas).  Neste sentido, acho que este tipo de questão
mais atrapalha (porque "era só para usar a fórmula") - a menos que,
justamente, se discuta *porque* falamos de multiplicidade: para que as
fórmulas fiquem mais simples (e você pode incluir "bonitas" também,
por minha conta).  Nada mais.  E esta "simplificação" do entendimento
através da simplificação das fórmulas não se justifica sempre: este
mesmo debate sobre multiplicidades leva a considerar objetos no
infinito (para que todas as retas se intersectem sempre em um ponto),
complexos (para x^2 + 1 = 0 ter raiz), etc.  Muitas vezes, é útil ter
esse entendimento unificado, onde tudo "só depende do grau".  Mas será
mesmo que se eu perguntar para você "em quantos pontos a reta x=3
corta a parábola y=x^2?" você vai dizer "2, é óbvio"?

Abraços,
-- 
Bernardo Freitas Paulo da Costa

-- 
Esta mensagem foi verificada pelo sistema de antiv�rus e
 acredita-se estar livre de perigo.


=
Instru��es para entrar na lista, sair da lista e usar a lista em
http://www.mat.puc-rio.br/~obmlistas/obm-l.html
=


Re: [obm-l] Re: [obm-l] Re: [obm-l] Re: [obm-l] Dúvida conceitual (equações)

2018-10-15 Por tôpico Claudio Buffara
Derivando e igualando a zero o lado esquerdo da sua equação, ficamos com:
-2*cos(x)*sen(x) + sen(x) = 0 ==>
sen(x) = 0  ou  cos(x) = 1/2 ==>
x = 0 ou x = pi ou x = 2pi
ou x = pi/3 ou x = 5pi/3.

Assim, uma definição que me parece adequado para equações em geral (e não 
necessariamente polinomiais) da forma f(x) = 0 é que uma raiz de multiplicidade 
n é raiz de f, f’, ... , f^(n-1) mas não é raiz de f^(n).

Naturalmente, se f não tiver todas as derivadas, precisaremos achar uma 
definição diferente. Mas talvez, neste caso, nem faça sentido falar em 
multiplicidade de uma raiz.

Enviado do meu iPhone

Em 15 de out de 2018, à(s) 08:13, Vanderlei Nemitz  
escreveu:

> Claudio:
> Eu ficaria com a mesma dúvida!
> Pensaria em apenas uma raiz.
> 
> Qual é a soma das raízes da equação (cos x)^2 - cos x + 1/4 = 0 no 
> intervalo [0, 2pi]?
> 
> Em seg, 15 de out de 2018 07:00, Claudio Buffara  
> escreveu:
>> Qual a soma das raizes de (2^x - 8)^3 = 0?
>> Se a equação acima fosse apresentada como:
>> 2^(3x) - 24*2^(2x) + 192*2^x - 512 = 0,
>> isso mudaria sua resposta?
>> 
>> Enviado do meu iPhone
>> 
>> Em 15 de out de 2018, Ã (s) 00:29, Vanderlei Nemitz  
>> escreveu:
>> 
>>> Valeu, Pedro! Tomara que mais alguém emita sua opinião.
>>> Um abraço!
>>> 
>>> Em dom, 14 de out de 2018 18:59, Pedro José  
>>> escreveu:
 Boa noite!
 Bom questionamento. Vou me posicionar na arquibancada. 
 Minha posição é controversa. Se quer se levar em conta a 
 repetição tem que se falar do produto das raízes, cada elevada a 
 sua multiplicidade. No caso de soma, cada raiz multiplicada pela 
 multiplicidade.
 Para esse exemplo, o conjunto solução é {1/2,-1} então o 
 produto é -1/2.
 Em suma, não aceito n raízes iguais, mas sim uma raiz de 
 multiplicidade n.
 Se quando queremos provar que algo é unico supomos a existência de 
 dois e provamos que são iguais. Creio que seja contraditório dois ou 
 nais iguais.
 Mas vamos observar as diversas posições, pois, creio que o assunto 
 não seja pacífico. 
 Saudações, 
 PJMS 
 
 Em Dom, 14 de out de 2018 06:33, Vanderlei Nemitz  
 escreveu:
> Bom dia!
> Na seguinte questão, que me foi apresentada por um aluno, a resposta 
> proposta é a alternativa C (1/2). Eu sempre pensei que apenas 
> considerávamos multiplicidades em equações polinomiais. Como 
> essa é uma equação exponencial, obtive a resposta B (-1/2). O 
> que é correto pensar?
> 
> O produto das raízes da equação 16.4^3x - 40.4^2x + 17.4^x - 2 = 
> 0 é igual a:
> A) 1
> B) - 0,5
> C) 0,5
> D) - 1
> E) 0
> 
> Muito obrigado!
> 
> -- 
> Esta mensagem foi verificada pelo sistema de antivírus e 
> acredita-se estar livre de perigo.
 
 -- 
 Esta mensagem foi verificada pelo sistema de antivírus e 
 acredita-se estar livre de perigo.
>>> 
>>> -- 
>>> Esta mensagem foi verificada pelo sistema de antivírus e 
>>> acredita-se estar livre de perigo.
>> 
>> -- 
>> Esta mensagem foi verificada pelo sistema de antivírus e 
>> acredita-se estar livre de perigo.
> 
> -- 
> Esta mensagem foi verificada pelo sistema de antivírus e 
> acredita-se estar livre de perigo.

-- 
Esta mensagem foi verificada pelo sistema de antiv�rus e
 acredita-se estar livre de perigo.



Re: [obm-l] Re: [obm-l] Re: [obm-l] Re: [obm-l] Dúvida conceitual (equações)

2018-10-15 Por tôpico Claudio Buffara
Pensando só como uma equação, talvez faça sentido não considerar a 
multiplicidade.

Mas, no seu exemplo, no intervalo [0,2pi], os gráficos de 
f(x) = cos(x) - 1/2 
e de
g(x) = (cos(x) - 1/2)^2
tem um comportamento bem distinto um do outro em vizinhanças de pi/3 e 5pi/3.
Por exemplo, o gráfico de f corta o eixo x em pi/3 enquanto que o de g apenas 
tangencia o eixo neste ponto.
Idem pros outros exemplos.
Isso sugere que, mesmo nestes casos, talvez seja conveniente considerar a 
multiplicidade de uma raiz.

Enviado do meu iPhone

Em 15 de out de 2018, à(s) 08:13, Vanderlei Nemitz  
escreveu:

> Claudio:
> Eu ficaria com a mesma dúvida!
> Pensaria em apenas uma raiz.
> 
> Qual é a soma das raízes da equação (cos x)^2 - cos x + 1/4 = 0 no 
> intervalo [0, 2pi]?
> 
> Em seg, 15 de out de 2018 07:00, Claudio Buffara  
> escreveu:
>> Qual a soma das raizes de (2^x - 8)^3 = 0?
>> Se a equação acima fosse apresentada como:
>> 2^(3x) - 24*2^(2x) + 192*2^x - 512 = 0,
>> isso mudaria sua resposta?
>> 
>> Enviado do meu iPhone
>> 
>> Em 15 de out de 2018, Ã (s) 00:29, Vanderlei Nemitz  
>> escreveu:
>> 
>>> Valeu, Pedro! Tomara que mais alguém emita sua opinião.
>>> Um abraço!
>>> 
>>> Em dom, 14 de out de 2018 18:59, Pedro José  
>>> escreveu:
 Boa noite!
 Bom questionamento. Vou me posicionar na arquibancada. 
 Minha posição é controversa. Se quer se levar em conta a 
 repetição tem que se falar do produto das raízes, cada elevada a 
 sua multiplicidade. No caso de soma, cada raiz multiplicada pela 
 multiplicidade.
 Para esse exemplo, o conjunto solução é {1/2,-1} então o 
 produto é -1/2.
 Em suma, não aceito n raízes iguais, mas sim uma raiz de 
 multiplicidade n.
 Se quando queremos provar que algo é unico supomos a existência de 
 dois e provamos que são iguais. Creio que seja contraditório dois ou 
 nais iguais.
 Mas vamos observar as diversas posições, pois, creio que o assunto 
 não seja pacífico. 
 Saudações, 
 PJMS 
 
 Em Dom, 14 de out de 2018 06:33, Vanderlei Nemitz  
 escreveu:
> Bom dia!
> Na seguinte questão, que me foi apresentada por um aluno, a resposta 
> proposta é a alternativa C (1/2). Eu sempre pensei que apenas 
> considerávamos multiplicidades em equações polinomiais. Como 
> essa é uma equação exponencial, obtive a resposta B (-1/2). O 
> que é correto pensar?
> 
> O produto das raízes da equação 16.4^3x - 40.4^2x + 17.4^x - 2 = 
> 0 é igual a:
> A) 1
> B) - 0,5
> C) 0,5
> D) - 1
> E) 0
> 
> Muito obrigado!
> 
> -- 
> Esta mensagem foi verificada pelo sistema de antivírus e 
> acredita-se estar livre de perigo.
 
 -- 
 Esta mensagem foi verificada pelo sistema de antivírus e 
 acredita-se estar livre de perigo.
>>> 
>>> -- 
>>> Esta mensagem foi verificada pelo sistema de antivírus e 
>>> acredita-se estar livre de perigo.
>> 
>> -- 
>> Esta mensagem foi verificada pelo sistema de antivírus e 
>> acredita-se estar livre de perigo.
> 
> -- 
> Esta mensagem foi verificada pelo sistema de antivírus e 
> acredita-se estar livre de perigo.

-- 
Esta mensagem foi verificada pelo sistema de antiv�rus e
 acredita-se estar livre de perigo.



[obm-l] Re: [obm-l] Re: [obm-l] Re: [obm-l] Dúvida conceitual (equações)

2018-10-15 Por tôpico Vanderlei Nemitz
Claudio:
Eu ficaria com a mesma dúvida!
Pensaria em apenas uma raiz.

Qual é a soma das raízes da equação (cos x)^2 - cos x + 1/4 = 0 no
intervalo [0, 2pi]?

Em seg, 15 de out de 2018 07:00, Claudio Buffara 
escreveu:

> Qual a soma das raizes de (2^x - 8)^3 = 0?
> Se a equação acima fosse apresentada como:
> 2^(3x) - 24*2^(2x) + 192*2^x - 512 = 0,
> isso mudaria sua resposta?
>
> Enviado do meu iPhone
>
> Em 15 de out de 2018, à(s) 00:29, Vanderlei Nemitz 
> escreveu:
>
> Valeu, Pedro! Tomara que mais alguém emita sua opinião.
> Um abraço!
>
> Em dom, 14 de out de 2018 18:59, Pedro José 
> escreveu:
>
>> Boa noite!
>> Bom questionamento. Vou me posicionar na arquibancada.Â
>> Minha posição é controversa. Se quer se levar em conta a repetição
>> tem que se falar do produto das raízes, cada elevada a sua multiplicidade.
>> No caso de soma, cada raiz multiplicada pela multiplicidade.
>> Para esse exemplo, o conjunto solução é {1/2,-1} então o produto é
>> -1/2.
>> Em suma, não aceito n raízes iguais, mas sim uma raiz de multiplicidade
>> n.
>> Se quando queremos provar que algo é unico supomos a existência de dois
>> e provamos que são iguais. Creio que seja contraditório dois ou nais
>> iguais.
>> Mas vamos observar as diversas posições, pois, creio que o assunto não
>> seja pacífico.Â
>> Saudações,Â
>> PJMSÂ
>>
>> Em Dom, 14 de out de 2018 06:33, Vanderlei Nemitz 
>> escreveu:
>>
>>> Bom dia!
>>> Na seguinte questão, que me foi apresentada por um aluno, a resposta
>>> proposta é a alternativa C (1/2). Eu sempre pensei que apenas
>>> considerávamos multiplicidades em equações polinomiais. Como essa é uma
>>> equação exponencial, obtive a resposta B (-1/2). O que é correto pensar?
>>>
>>> O produto das raízes da equação 16.4^3x - 40.4^2x + 17.4^x - 2 = 0 é
>>> igual a:
>>> A) 1
>>> B) - 0,5
>>> C) 0,5
>>> D) - 1
>>> E) 0
>>>
>>> Muito obrigado!
>>>
>>> --
>>> Esta mensagem foi verificada pelo sistema de antivírus e
>>> acredita-se estar livre de perigo.
>>
>>
>> --
>> Esta mensagem foi verificada pelo sistema de antivírus e
>> acredita-se estar livre de perigo.
>
>
> --
> Esta mensagem foi verificada pelo sistema de antivírus e
> acredita-se estar livre de perigo.
>
>
> --
> Esta mensagem foi verificada pelo sistema de antivírus e
> acredita-se estar livre de perigo.
>

-- 
Esta mensagem foi verificada pelo sistema de antiv�rus e
 acredita-se estar livre de perigo.



[obm-l] Re: [obm-l] Re: [obm-l] Re: [obm-l] Re: [obm-l] Dúvida num Enunciado

2018-04-26 Por tôpico Pedro José
Boa tarde!
Bernardo,
Realmente eu falhei. Fiquei com a expressão |x+3| < 4 na cabeça. Até uso um
delta, e comento que não pode ser maior que 4.
Saudações,
PJMS

Em 25 de abr de 2018 22:33, "Jaare Oregim" 
escreveu:

>
>
> 2018-04-25 21:30 GMT-03:00 Bernardo Freitas Paulo da Costa <
> bernardo...@gmail.com>:
>
>> 2018-04-25 20:41 GMT-03:00 Claudio Buffara :
>> > O [...]
>> "Determine r > 0 tal que [ |x+3| < r => (A^2 - 10A + 9 > 0 para todo A
>> real) ]."
>>
>> Que continua com o "problema" de ter um "x" livre.  Daí, a proposição
>> entre colchetes tem um valor (verdadeiro/falso) que depende de x.
>>
>
> se o x tá livre *não* tem valor-verdade. Sentença aberta não tem
> valor-verdade.
>
> tb acho que a intenção é "Determine r > 0 tal que (para todo x real, |x+3|
> < r => x^2 - 10x + 9 > 0)."
>
> --
> Esta mensagem foi verificada pelo sistema de antivírus e
> acredita-se estar livre de perigo.

-- 
Esta mensagem foi verificada pelo sistema de antiv�rus e
 acredita-se estar livre de perigo.



[obm-l] Re: [obm-l] Re: [obm-l] Re: [obm-l] Dúvida num Enunciado

2018-04-25 Por tôpico Jaare Oregim
2018-04-25 21:30 GMT-03:00 Bernardo Freitas Paulo da Costa <
bernardo...@gmail.com>:

> 2018-04-25 20:41 GMT-03:00 Claudio Buffara :
> > O [...]
> "Determine r > 0 tal que [ |x+3| < r => (A^2 - 10A + 9 > 0 para todo A
> real) ]."
>
> Que continua com o "problema" de ter um "x" livre.  Daí, a proposição
> entre colchetes tem um valor (verdadeiro/falso) que depende de x.
>

se o x tá livre *não* tem valor-verdade. Sentença aberta não tem
valor-verdade.

tb acho que a intenção é "Determine r > 0 tal que (para todo x real, |x+3|
< r => x^2 - 10x + 9 > 0)."

-- 
Esta mensagem foi verificada pelo sistema de antiv�rus e
 acredita-se estar livre de perigo.



[obm-l] Re: [obm-l] Re: [obm-l] Re: [obm-l] Re: [obm-l] Dúvida num Enunciado

2018-04-25 Por tôpico Claudio Buffara
Verdade! Reparei agora que deve ser r > 0.
Então provavelmente o "para todo x real" não deveria estar lá.
Neste caso, vira um problema com mais cara de EM:

Achar todos os r > 0 tais que
SE  x pertence ao intervalo (-3-r , -3+r )
ENTÃO  x^2 - 10x + 9 > 0

x^2 - 10x + 9 > 0  sss  x pertence a (-inf,1) união (9,+inf).

Assim, observamos que se 0 < r <= 4, então (-3-r,-3+r) está contido em
(-inf,1).
Logo, se 0 < r <= 4 então a implicação acima é verdadeira.

[]s,
Claudio.




2018-04-25 21:47 GMT-03:00 Pedro José :

> Boa noite!
> Cláudio,
> o problema tem restrição r>0. Não dá para seguir nessa linha de r< 0.
> Saudações,
> PJMS
>
> Em 25 de abr de 2018 21:42, "Bernardo Freitas Paulo da Costa" <
> bernardo...@gmail.com> escreveu:
>
>> 2018-04-25 20:20 GMT-03:00 Pedro José :
>> > Boa tarde!
>> > Realmente o enunciado está mal feito.
>> >
>> > Se |x+3| < r, não pode ser para todo o Real. Na verdade é x pertence a
>> |R.
>> >
>> > x^2 -10x + 9 >0  ==> x pertence a A = (-oo, 1) U (9,oo)
>> >
>> > então temos que escolher r de modo que quando resolvamos |x + 3| < r,
>> tenha
>> > x num subconjunto de A
>> >
>> > x < -3 ==> x+3 < 0 ==> -x -3 < r ==> r > x+3 Se r > 4 vai ter 1=< x =<9
>> > atendendo |x +3| <4 + delta. Portanto x <4
>> > então |x+3| < 4, conferindo
>> > x > -3 ==> x+3 <4  ==> x<1, atende.
>> > se x<-3 atende por hipótese. Mas se quiser conferir. -x - 3 < 4 : -x <
>> 7: x
>> >>7, mas x <-3, não tem solução.
>> >
>> > x>=- 3 ==> x+3>=0 ==> x+3 < r. Se r >=4, existirá solução em [1,9].
>> >
>> > Portanto r pertence a (0,4)
>>
>> Só um detalhe: r = 4 também serve: se |x+3| < 4, temos -7 < x < 1, que
>> está contido em A.
>>
>> A minha forma preferida de resolver este exercício é gráfica:
>> desenhamos o conjunto A, depois tomamos P = -3, e traçamos um
>> intervalo simétrico em P de maior raio possível contido em A.  Dá r <=
>> 4 ou r < 4 (no desenho, é difícil decidir entre o estrito ou não) e
>> daí tem que pensar um pouco para detectar se r = 4 serve.
>>
>> Abraços,
>> --
>> Bernardo Freitas Paulo da Costa
>>
>> --
>> Esta mensagem foi verificada pelo sistema de antivírus e
>>  acredita-se estar livre de perigo.
>>
>>
>> =
>> Instru�ões para entrar na lista, sair da lista e usar a lista em
>> http://www.mat.puc-rio.br/~obmlistas/obm-l.html
>> =
>>
>
> --
> Esta mensagem foi verificada pelo sistema de antivírus e
> acredita-se estar livre de perigo.
>

-- 
Esta mensagem foi verificada pelo sistema de antiv�rus e
 acredita-se estar livre de perigo.



[obm-l] Re: [obm-l] Re: [obm-l] Re: [obm-l] Re: [obm-l] Dúvida num Enunciado

2018-04-25 Por tôpico Luiz Antonio Rodrigues
Olá, Bernardo!
Boa noite!
Vou tentar fazer a resolução graficamente...
Muito obrigado!
Um abraço!
Luiz

On Wed, Apr 25, 2018, 9:55 PM Pedro José  wrote:

> Boa noite!
> Cláudio,
> o problema tem restrição r>0. Não dá para seguir nessa linha de r< 0.
> Saudações,
> PJMS
>
> Em 25 de abr de 2018 21:42, "Bernardo Freitas Paulo da Costa" <
> bernardo...@gmail.com> escreveu:
>
>> 2018-04-25 20:20 GMT-03:00 Pedro José :
>> > Boa tarde!
>> > Realmente o enunciado está mal feito.
>> >
>> > Se |x+3| < r, não pode ser para todo o Real. Na verdade é x pertence a
>> |R.
>> >
>> > x^2 -10x + 9 >0  ==> x pertence a A = (-oo, 1) U (9,oo)
>> >
>> > então temos que escolher r de modo que quando resolvamos |x + 3| < r,
>> tenha
>> > x num subconjunto de A
>> >
>> > x < -3 ==> x+3 < 0 ==> -x -3 < r ==> r > x+3 Se r > 4 vai ter 1=< x =<9
>> > atendendo |x +3| <4 + delta. Portanto x <4
>> > então |x+3| < 4, conferindo
>> > x > -3 ==> x+3 <4  ==> x<1, atende.
>> > se x<-3 atende por hipótese. Mas se quiser conferir. -x - 3 < 4 : -x <
>> 7: x
>> >>7, mas x <-3, não tem solução.
>> >
>> > x>=- 3 ==> x+3>=0 ==> x+3 < r. Se r >=4, existirá solução em [1,9].
>> >
>> > Portanto r pertence a (0,4)
>>
>> Só um detalhe: r = 4 também serve: se |x+3| < 4, temos -7 < x < 1, que
>> está contido em A.
>>
>> A minha forma preferida de resolver este exercício é gráfica:
>> desenhamos o conjunto A, depois tomamos P = -3, e traçamos um
>> intervalo simétrico em P de maior raio possível contido em A.  Dá r <=
>> 4 ou r < 4 (no desenho, é difícil decidir entre o estrito ou não) e
>> daí tem que pensar um pouco para detectar se r = 4 serve.
>>
>> Abraços,
>> --
>> Bernardo Freitas Paulo da Costa
>>
>> --
>> Esta mensagem foi verificada pelo sistema de antivírus e
>>  acredita-se estar livre de perigo.
>>
>>
>> =
>> Instru�ões para entrar na lista, sair da lista e usar a lista em
>> http://www.mat.puc-rio.br/~obmlistas/obm-l.html
>> =
>>
>
> --
> Esta mensagem foi verificada pelo sistema de antivírus e
> acredita-se estar livre de perigo.

-- 
Esta mensagem foi verificada pelo sistema de antiv�rus e
 acredita-se estar livre de perigo.



[obm-l] Re: [obm-l] Re: [obm-l] Re: [obm-l] Dúvida num Enunciado

2018-04-25 Por tôpico Pedro José
Boa noite!
Cláudio,
o problema tem restrição r>0. Não dá para seguir nessa linha de r< 0.
Saudações,
PJMS

Em 25 de abr de 2018 21:42, "Bernardo Freitas Paulo da Costa" <
bernardo...@gmail.com> escreveu:

> 2018-04-25 20:20 GMT-03:00 Pedro José :
> > Boa tarde!
> > Realmente o enunciado está mal feito.
> >
> > Se |x+3| < r, não pode ser para todo o Real. Na verdade é x pertence a
> |R.
> >
> > x^2 -10x + 9 >0  ==> x pertence a A = (-oo, 1) U (9,oo)
> >
> > então temos que escolher r de modo que quando resolvamos |x + 3| < r,
> tenha
> > x num subconjunto de A
> >
> > x < -3 ==> x+3 < 0 ==> -x -3 < r ==> r > x+3 Se r > 4 vai ter 1=< x =<9
> > atendendo |x +3| <4 + delta. Portanto x <4
> > então |x+3| < 4, conferindo
> > x > -3 ==> x+3 <4  ==> x<1, atende.
> > se x<-3 atende por hipótese. Mas se quiser conferir. -x - 3 < 4 : -x <
> 7: x
> >>7, mas x <-3, não tem solução.
> >
> > x>=- 3 ==> x+3>=0 ==> x+3 < r. Se r >=4, existirá solução em [1,9].
> >
> > Portanto r pertence a (0,4)
>
> Só um detalhe: r = 4 também serve: se |x+3| < 4, temos -7 < x < 1, que
> está contido em A.
>
> A minha forma preferida de resolver este exercício é gráfica:
> desenhamos o conjunto A, depois tomamos P = -3, e traçamos um
> intervalo simétrico em P de maior raio possível contido em A.  Dá r <=
> 4 ou r < 4 (no desenho, é difícil decidir entre o estrito ou não) e
> daí tem que pensar um pouco para detectar se r = 4 serve.
>
> Abraços,
> --
> Bernardo Freitas Paulo da Costa
>
> --
> Esta mensagem foi verificada pelo sistema de antivírus e
>  acredita-se estar livre de perigo.
>
>
> =
> Instru�ões para entrar na lista, sair da lista e usar a lista em
> http://www.mat.puc-rio.br/~obmlistas/obm-l.html
> =
>

-- 
Esta mensagem foi verificada pelo sistema de antiv�rus e
 acredita-se estar livre de perigo.



[obm-l] Re: [obm-l] Re: [obm-l] Re: [obm-l] Re: [obm-l] Dúvida em uma solução (conjunto denso)

2017-07-10 Por tôpico Pedro Soares
Sim, é uma prova por absurdo.

''...o autor parte de uma hipótese contrária ao resultado pra chegar num
absurdo...''

2017-07-11 1:03 GMT-03:00 Bernardo Freitas Paulo da Costa <
bernardo...@gmail.com>:

> 2017-07-10 18:56 GMT+03:00 Antonio Carlos :
> > Entendi. Muito obrigado, Pedro!
>
> Tem um problema muito sério, que os logs são diferentes...
>
> log_2 3 = log(3)/log(2) = 1.5849625007211563
> log_3 6 = log(6)/log(3) = 1.6309297535714573
>
> Mas o problema está, provavelmente, na primeira hipótese (que ela
> também é falsa).  A demonstração por densidade está certa, e talvez
> seja no meio de um raciocínio por absurdo, mas sei lá...
> --
> Bernardo Freitas Paulo da Costa
>
> --
> Esta mensagem foi verificada pelo sistema de antivírus e
>  acredita-se estar livre de perigo.
>
>
> =
> Instru�ões para entrar na lista, sair da lista e usar a lista em
> http://www.mat.puc-rio.br/~obmlistas/obm-l.html
> =
>

-- 
Esta mensagem foi verificada pelo sistema de antiv�rus e
 acredita-se estar livre de perigo.



[obm-l] Re: [obm-l] Re: [obm-l] Re: [obm-l] Dúvida em uma solução (conjunto denso)

2017-07-10 Por tôpico Bernardo Freitas Paulo da Costa
2017-07-10 18:56 GMT+03:00 Antonio Carlos :
> Entendi. Muito obrigado, Pedro!

Tem um problema muito sério, que os logs são diferentes...

log_2 3 = log(3)/log(2) = 1.5849625007211563
log_3 6 = log(6)/log(3) = 1.6309297535714573

Mas o problema está, provavelmente, na primeira hipótese (que ela
também é falsa).  A demonstração por densidade está certa, e talvez
seja no meio de um raciocínio por absurdo, mas sei lá...
-- 
Bernardo Freitas Paulo da Costa

-- 
Esta mensagem foi verificada pelo sistema de antiv�rus e
 acredita-se estar livre de perigo.


=
Instru��es para entrar na lista, sair da lista e usar a lista em
http://www.mat.puc-rio.br/~obmlistas/obm-l.html
=


[obm-l] Re: [obm-l] Re: [obm-l] Re: [obm-l] Re: [obm-l] Dúvida sobre a Obm U

2016-07-26 Por tôpico Carlos Gomes
Um bom livro é Razvan Gelca, Titu Andreescu-Putnam and Beyond (2007)

Cgomes.

Em 26 de julho de 2016 08:57, Otávio Araújo 
escreveu:

> Não, onde posso conseguir? e do que ela trata?
>
> Em 25 de julho de 2016 11:32, Carlos Victor 
> escreveu:
>
>>
>>
>>
>> Oi Otávio,
>>
>> Você já viu a Revista Matemática Universitária da SBM ?
>>
>> Em 25/07/2016 10:09, Otávio Araújo escreveu:
>>
>>
>>
>> Pois é, se algum professor com experiência em olimpíadas, como o Nicolau
>> por exemplo, respondesse minha pergunta seria de grande ajuda
>>
>> Em 24 de jul de 2016, às 23:25, Israel Meireles Chrisostomo <
>> israelmchrisost...@gmail.com> escreveu:
>>
>> Boa pergunta, eu também tenho interesse em participar da OBM U e
>> gostaria de umas dicas
>>
>> Em 16 de julho de 2016 13:29, Otávio Araújo 
>> escreveu:
>>
>>> Galera, gostaria que vocês me dessem dicas de o que estudar, como
>>> estudar e por quais livros e materiais estudar para a prova da Obm nível
>>> universitário...
>>> Estou muito interessado em participar, mas fico meio confuso por onde
>>> estudar...
>>> Por favor me ajudem
>>> --
>>> Esta mensagem foi verificada pelo sistema de antivírus e
>>> Â acredita-se estar livre de perigo.
>>>
>>>
>>> =
>>> Instruções para entrar na lista, sair da lista e usar a lista em
>>> http://www.mat.puc-rio.br/~obmlistas/obm-l.html
>>> =
>>>
>>
>> --
>> Esta mensagem foi verificada pelo sistema de antivírus e
>> acredita-se estar livre de perigo.
>>
>>
>> --
>> Esta mensagem foi verificada pelo sistema de antivrus e
>> acredita-se estar livre de perigo.
>>
>>
>> --
>> Esta mensagem foi verificada pelo sistema de antivírus e
>> acredita-se estar livre de perigo.
>>
>
>
> --
> Esta mensagem foi verificada pelo sistema de antivírus e
> acredita-se estar livre de perigo.
>

-- 
Esta mensagem foi verificada pelo sistema de antiv�rus e
 acredita-se estar livre de perigo.



[obm-l] Re: [obm-l] Re: [obm-l] Re: [obm-l] Dúvida sobre a Obm U

2016-07-26 Por tôpico Otávio Araújo
Não, onde posso conseguir? e do que ela trata?

Em 25 de julho de 2016 11:32, Carlos Victor 
escreveu:

>
>
>
> Oi Otávio,
>
> Você já viu a Revista Matemática Universitária da SBM ?
>
> Em 25/07/2016 10:09, Otávio Araújo escreveu:
>
>
>
> Pois é, se algum professor com experiência em olimpíadas, como o Nicolau
> por exemplo, respondesse minha pergunta seria de grande ajuda
>
> Em 24 de jul de 2016, às 23:25, Israel Meireles Chrisostomo <
> israelmchrisost...@gmail.com> escreveu:
>
> Boa pergunta, eu também tenho interesse em participar da OBM U e gostaria
> de umas dicas
>
> Em 16 de julho de 2016 13:29, Otávio Araújo 
> escreveu:
>
>> Galera, gostaria que vocês me dessem dicas de o que estudar, como
>> estudar e por quais livros e materiais estudar para a prova da Obm nível
>> universitário...
>> Estou muito interessado em participar, mas fico meio confuso por onde
>> estudar...
>> Por favor me ajudem
>> --
>> Esta mensagem foi verificada pelo sistema de antivírus e
>> Â acredita-se estar livre de perigo.
>>
>>
>> =
>> Instruções para entrar na lista, sair da lista e usar a lista em
>> http://www.mat.puc-rio.br/~obmlistas/obm-l.html
>> =
>>
>
> --
> Esta mensagem foi verificada pelo sistema de antivírus e
> acredita-se estar livre de perigo.
>
>
> --
> Esta mensagem foi verificada pelo sistema de antivrus e
> acredita-se estar livre de perigo.
>
>
> --
> Esta mensagem foi verificada pelo sistema de antivírus e
> acredita-se estar livre de perigo.
>

-- 
Esta mensagem foi verificada pelo sistema de antiv�rus e
 acredita-se estar livre de perigo.



[obm-l] Re: [obm-l] Re: [obm-l] Re: [obm-l] Re: [obm-l] Dúvida sobre a Obm U

2016-07-25 Por tôpico Tiago Sandino
Égua ma, sou mais ou menos da UFC, de qualquer forma, começar matemática
UFC prox ano. Fiz olimpíada um tempo, imergi totalmente nisso. Fiz e
trabalhei com engenharia elétrica uns anos, larguei o curso no final pq o
negócio na engenharia era próprio e precisava de tempo. Atualmente tô dando
aula de turma ITA IME e olimpíada de mat no Colégio Militar. Não sei se eu
vou poder fazer a prova da OBMU, mas estudarei. E a olimpíada universitária
é uma continuação da não universitária, daí meu interesse, já que sou
professor disso.
Precisando de dica, ta aí meu contato. 85 9 99134896. Se tem uma coisa que
eu sei sobre olimpíada é que o cara tem que tá no meio, tem que falar sobre
isso, tem que conhecer pessoas do meio etc. Só assim vc evolui. Só assim vc
passa de ser um cara que consegue aplicar fórmula a ser um cara que "cria"
matemática todo dia, que na minha concepção é a maior realização pessoal na
olimpíada.


Em 25 de julho de 2016 17:19, Otávio Araújo 
escreveu:

>
>
> Égua Tiago, eu também sou do Ceará mas meu celular atualmente não tem
> chip Mas tu é da UFC Tiago? E ainda estou esperando algum professor com
> experiência em olimpíadas de matemática responder a minha pergunta
>
> Em 25 de jul de 2016, às 13:38, Tiago Sandino 
> escreveu:
>
> Oi pessoal.
> Tem diversos livros de olimpíadas para graduandos (undergrads) ou com
> capítulos de temas exclusivamente (até onde eu saiba) universitários.
> Grátis na net, que eu saiba, tem muita coisa no AOPS. Dois links aqui:
> 1) *Fórum*: https://www.artofproblemsolving.com/community/c7_college_math
> 2) *Fórum por Competições*:
> https://www.artofproblemsolving.com/community/c15_undergraduate_contests
>
> Sou do Ceará, tava meio afastado da Matemática, mas fiz as pazes com ela
> recentemente. Se alguém quiser formar um grupo de estudo pelo Whatsapp...
> segue meu número: 85 9 9913 4896.
>
> Att.
> Tiago Sandino
>
> Em 25 de julho de 2016 10:20, Raul Alves  escreveu:
>
>> Também tenho interesse na OBMU, e a 1ª fase tá chegando.
>> Se algum professor puder organizar algum material de apoio, seria de
>> grande ajuda
>>
>> Em 25 de julho de 2016 10:09, Otávio Araújo 
>> escreveu:
>>
>>>
>>>
>>> Pois é, se algum professor com experiência em olimpíadas, como o
>>> Nicolau por exemplo, respondesse minha pergunta seria de grande ajuda
>>>
>>> Em 24 de jul de 2016, Ã s 23:25, Israel Meireles Chrisostomo <
>>> israelmchrisost...@gmail.com> escreveu:
>>>
>>> Boa pergunta, eu também tenho interesse em participar da OBM U e
>>> gostaria de umas dicas
>>>
>>> Em 16 de julho de 2016 13:29, Otávio Araújo <
>>> otavio17.ara...@gmail.com> escreveu:
>>>
 Galera, gostaria que vocês me dessem dicas de o que estudar,
 como estudar e por quais livros e materiais estudar para a prova da Obm
 nível universitário...
 Estou muito interessado em participar, mas fico meio confuso por onde
 estudar...
 Por favor me ajudem
 --
 Esta mensagem foi verificada pelo sistema de antivírus e
  acredita-se estar livre de perigo.



 =
 Instruções para entrar na lista, sair da lista e usar a lista em
 http://www.mat.puc-rio.br/~obmlistas/obm-l.html

 =

>>>
>>>
>>> --
>>> Esta mensagem foi verificada pelo sistema de antivírus e
>>> acredita-se estar livre de perigo.
>>>
>>>
>>> --
>>> Esta mensagem foi verificada pelo sistema de antivírus e
>>> acredita-se estar livre de perigo.
>>>
>>
>>
>>
>> --
>> *Raul Lima Alves*
>>
>> *Estagiário na Aton Engenharia*
>> *Estudante de Engenharia de Computação - UFBA*
>> *Telefone: (71) 9103-0878*
>> *Facebook:Â *https://www.facebook.com/raul.alves.161
>> *LinkedIn*:Â https://br.linkedin.com/in/raul-alves-8b090228
>> 
>>
>>
>>
>> --
>> Esta mensagem foi verificada pelo sistema de antivírus e
>> acredita-se estar livre de perigo.
>>
>
>
> --
> Esta mensagem foi verificada pelo sistema de antivírus e
> acredita-se estar livre de perigo.
>
>
> --
> Esta mensagem foi verificada pelo sistema de antivírus e
> acredita-se estar livre de perigo.
>

-- 
Esta mensagem foi verificada pelo sistema de antiv�rus e
 acredita-se estar livre de perigo.



Re: [obm-l] Re: [obm-l] Re: [obm-l] Re: [obm-l] Dúvida sobre a Obm U

2016-07-25 Por tôpico Otávio Araújo


Égua Tiago, eu também sou do Ceará mas meu celular atualmente não tem chip 
Mas tu é da UFC Tiago? E ainda estou esperando algum professor com experiência 
em olimpíadas de matemática responder a minha pergunta 

> Em 25 de jul de 2016, às 13:38, Tiago Sandino  
> escreveu:
> 
> Oi pessoal.
> Tem diversos livros de olimpíadas para graduandos (undergrads) ou com 
> capítulos de temas exclusivamente (até onde eu saiba) universitários. 
> Grátis na net, que eu saiba, tem muita coisa no AOPS. Dois links aqui:
> 1) Fórum: https://www.artofproblemsolving.com/community/c7_college_math
> 2) Fórum por Competições: 
> https://www.artofproblemsolving.com/community/c15_undergraduate_contests
> 
> Sou do Ceará, tava meio afastado da Matemática, mas fiz as pazes com ela 
> recentemente. Se alguém quiser formar um grupo de estudo pelo Whatsapp... 
> segue meu número: 85 9 9913 4896.
> 
> Att.
> Tiago Sandino
> 
> Em 25 de julho de 2016 10:20, Raul Alves  escreveu:
>> Também tenho interesse na OBMU, e a 1ª fase tá chegando.
>> Se algum professor puder organizar algum material de apoio, seria de grande 
>> ajuda
>> 
>> Em 25 de julho de 2016 10:09, Otávio Araújo  
>> escreveu:
>>> 
>>> 
>>> Pois é, se algum professor com experiência em olimpíadas, como o Nicolau 
>>> por exemplo, respondesse minha pergunta seria de grande ajuda
>>> 
>>> Em 24 de jul de 2016, Ã s 23:25, Israel Meireles Chrisostomo 
>>>  escreveu:
>>> 
 Boa pergunta, eu também tenho interesse em participar da OBM U e 
 gostaria de umas dicas
 
 Em 16 de julho de 2016 13:29, Otávio Araújo 
  escreveu:
> Galera, gostaria que vocês me dessem dicas de o que estudar, como 
> estudar e por quais livros e materiais estudar para a prova da Obm 
> nível universitário...
> Estou muito interessado em participar, mas fico meio confuso por onde 
> estudar...
> Por favor me ajudem
> --
> Esta mensagem foi verificada pelo sistema de antivírus e
>  acredita-se estar livre de perigo.
> 
> 
> =
> Instruções para entrar na lista, sair da lista e usar a lista em
> http://www.mat.puc-rio.br/~obmlistas/obm-l.html
> =
 
 
 -- 
 Esta mensagem foi verificada pelo sistema de antivírus e 
 acredita-se estar livre de perigo.
>>> 
>>> -- 
>>> Esta mensagem foi verificada pelo sistema de antivírus e 
>>> acredita-se estar livre de perigo.
>> 
>> 
>> 
>> -- 
>> Raul Lima Alves
>> 
>> Estagiário na Aton Engenharia
>> Estudante de Engenharia de Computação - UFBA
>> Telefone: (71) 9103-0878
>> Facebook:Â https://www.facebook.com/raul.alves.161
>> LinkedIn:Â https://br.linkedin.com/in/raul-alves-8b090228
>> 
>> 
>> 
>> -- 
>> Esta mensagem foi verificada pelo sistema de antivírus e 
>> acredita-se estar livre de perigo.
> 
> 
> -- 
> Esta mensagem foi verificada pelo sistema de antivírus e 
> acredita-se estar livre de perigo.

-- 
Esta mensagem foi verificada pelo sistema de antiv�rus e
 acredita-se estar livre de perigo.



[obm-l] Re: [obm-l] Re: [obm-l] Re: [obm-l] Dúvida sobre a Obm U

2016-07-25 Por tôpico Tiago Sandino
Oi pessoal.
Tem diversos livros de olimpíadas para graduandos (undergrads) ou com
capítulos de temas exclusivamente (até onde eu saiba) universitários.
Grátis na net, que eu saiba, tem muita coisa no AOPS. Dois links aqui:
1) *Fórum*: https://www.artofproblemsolving.com/community/c7_college_math
2) *Fórum por Competições*:
https://www.artofproblemsolving.com/community/c15_undergraduate_contests

Sou do Ceará, tava meio afastado da Matemática, mas fiz as pazes com ela
recentemente. Se alguém quiser formar um grupo de estudo pelo Whatsapp...
segue meu número: 85 9 9913 4896.

Att.
Tiago Sandino

Em 25 de julho de 2016 10:20, Raul Alves  escreveu:

> Também tenho interesse na OBMU, e a 1ª fase tá chegando.
> Se algum professor puder organizar algum material de apoio, seria de
> grande ajuda
>
> Em 25 de julho de 2016 10:09, Otávio Araújo 
> escreveu:
>
>>
>>
>> Pois é, se algum professor com experiência em olimpíadas, como o Nicolau
>> por exemplo, respondesse minha pergunta seria de grande ajuda
>>
>> Em 24 de jul de 2016, às 23:25, Israel Meireles Chrisostomo <
>> israelmchrisost...@gmail.com> escreveu:
>>
>> Boa pergunta, eu também tenho interesse em participar da OBM U e
>> gostaria de umas dicas
>>
>> Em 16 de julho de 2016 13:29, Otávio Araújo 
>> escreveu:
>>
>>> Galera, gostaria que vocês me dessem dicas de o que estudar, como
>>> estudar e por quais livros e materiais estudar para a prova da Obm nível
>>> universitário...
>>> Estou muito interessado em participar, mas fico meio confuso por onde
>>> estudar...
>>> Por favor me ajudem
>>> --
>>> Esta mensagem foi verificada pelo sistema de antivírus e
>>> Â acredita-se estar livre de perigo.
>>>
>>>
>>> =
>>> Instruções para entrar na lista, sair da lista e usar a lista em
>>> http://www.mat.puc-rio.br/~obmlistas/obm-l.html
>>> =
>>>
>>
>>
>> --
>> Esta mensagem foi verificada pelo sistema de antivírus e
>> acredita-se estar livre de perigo.
>>
>>
>> --
>> Esta mensagem foi verificada pelo sistema de antivírus e
>> acredita-se estar livre de perigo.
>>
>
>
>
> --
> *Raul Lima Alves*
>
> *Estagiário na Aton Engenharia*
> *Estudante de Engenharia de Computação - UFBA*
> *Telefone: (71) 9103-0878*
> *Facebook: *https://www.facebook.com/raul.alves.161
> *LinkedIn*: https://br.linkedin.com/in/raul-alves-8b090228
> 
>
>
>
> --
> Esta mensagem foi verificada pelo sistema de antivírus e
> acredita-se estar livre de perigo.
>

-- 
Esta mensagem foi verificada pelo sistema de antiv�rus e
 acredita-se estar livre de perigo.



[obm-l] Re: [obm-l] Re: [obm-l] Re: [obm-l] Dúvida em Geometria Plana

2016-06-05 Por tôpico Carlos Gomes
De nada amigo! Sempre um prazer qdo posso ajudar!

Abraço, Cgomes.

Em 2 de junho de 2016 19:03, Daniel Rocha 
escreveu:

> Muito Obrigado, Carlos !!!
>
> Em 2 de junho de 2016 18:54, Carlos Gomes  escreveu:
>
>> Seja x a medida do ângulo BAC. Como o triângulo APQ é isosceles de base
>> AP, segue q a medida do ângulo APQ também é x. Note que o ângulo BQP é
>> externo ao triângulo APQ, portanto, mede x+x=2x. Agora como o triângulo BQP
>> é isosceles de base BQ, segue que o ângulo PBQ também mede 2x. Por fim note
>> que o ângulo BPC é externo ao triângulo  ABP, portanto mede x+2x=3x...como
>> o triângulo BCP também é isosceles de base PC, segue que o ângulo PCB
>> também mede 3x...como o triângulo ABC é isosceles, segue que o ângulo ABC
>> também mede 3x, o que revela q o ângulo PCB mede x. Assim, no triângulo BCP
>> temos que
>> x+3x+3x=π   ==>x=π/7.
>> Em 2 de jun de 2016 18:32, "Daniel Rocha" 
>> escreveu:
>>
>>> Olá a todos,
>>>
>>> Alguém poderia, por favor, apresentar os cálculos corretos da seguinte
>>> questão:
>>>
>>> Considere um triângulo ABC isósceles de base BC, e os pontos P e Q tais
>>> que P pertence a AC e Q pertence a AB. Se BC=BP=PQ=QA, a medida do ângulo
>>> do vértice A, em radianos, é:
>>>
>>> GABARITO: Pi/7.
>>>
>>> Eu agradeço a quem apresentar os cálculos corretos.
>>>
>>> --
>>> Esta mensagem foi verificada pelo sistema de antivírus e
>>> acredita-se estar livre de perigo.
>>
>>
>> --
>> Esta mensagem foi verificada pelo sistema de antivírus e
>> acredita-se estar livre de perigo.
>
>
>
> --
> Esta mensagem foi verificada pelo sistema de antivírus e
> acredita-se estar livre de perigo.
>

-- 
Esta mensagem foi verificada pelo sistema de antiv�rus e
 acredita-se estar livre de perigo.



[obm-l] Re: [obm-l] Re: [obm-l] Re: [obm-l] dúvida

2015-07-09 Por tôpico Israel Meireles Chrisostomo
Obrigado Ralph

Em 9 de julho de 2015 12:37, Ralph Teixeira  escreveu:

> Vamos generalizar para R^n: com a noção usual (Euclideana) de comprimento,
> o comprimento do segmento que liga (x1,x2,...,xn) a (y1,y2,...,yn) é:
>
> d=raiz((y1-x1)^2+(y2-x2)^2+...+(yn-xn)^2)
>
> Esta é a noção usual de distância entre dois pontos -- confira que é o que
> você conhece na reta (n=1) e no plano (n=2).
>
> Abraço, Ralph.
>
> 2015-07-09 10:27 GMT-03:00 Pedro José :
>
>> Bom dia!
>>
>> E o segmento???
>>
>> Em 8 de julho de 2015 21:48, Israel Meireles Chrisostomo <
>> israelmchrisost...@gmail.com> escreveu:
>>
>>> Como posso encontrar o comprimento de um segmento de reta no espaço
>>> tridimensional?Considere a origem da reta no ponto (x_0,y_0,z_0) e o final
>>> da reta no ponto (x_1,y_1,z_1)
>>>
>>> --
>>> Esta mensagem foi verificada pelo sistema de antivírus e
>>> acredita-se estar livre de perigo.
>>
>>
>>
>> --
>> Esta mensagem foi verificada pelo sistema de antivírus e
>> acredita-se estar livre de perigo.
>
>
>
> --
> Esta mensagem foi verificada pelo sistema de antivírus e
> acredita-se estar livre de perigo.
>

-- 
Esta mensagem foi verificada pelo sistema de antiv�rus e
 acredita-se estar livre de perigo.



[obm-l] Re: [obm-l] RE: [obm-l] RE: [obm-l] Dúvida Indução

2012-05-17 Por tôpico Ralph Teixeira
Cuidado: ao passar de n=k para n=k+1 no Passo de Inducao... o ultimo
termo "era" 3n-1, agora eh 3(n+1)-1=3n+2 -- nao eh questao de "somar
um no termo", eh "trocar n por n+1".

Abraco,
 Ralph

2012/5/17 Thiago Bersch :
> Então eu estava tentando fazer mas parava no mesmo ponto, fazia
> 2+5+8+...+(3n-1)+[(3n-1)+1], chegando aí eu me perco
>
> 
> From: joao_maldona...@hotmail.com
> To: obm-l@mat.puc-rio.br
> Subject: [obm-l] RE: [obm-l] Dúvida Indução
> Date: Mon, 14 May 2012 15:24:47 -0300
>
> Vamos dizer que para n respeite a formula
> Logo 2+4+6+...+2n=n.(n+1)
> Somando 2n+2
> 2+4+6+...+(2n+2=n(n+1)+2n+2=(n+1)(n+2) que respeita a formula
> Logo se vale para n, vale para n+1
> Como vale para 1, vale para 2, e  entao para 3, 4, 5...
> Vale para qualquer natural
>
> Tente fazer o segundo agora
> []s Joao
>
> 
> From: thiago_t...@hotmail.com
> To: obm-l@mat.puc-rio.br
> Subject: [obm-l] Dúvida Indução
> Date: Mon, 14 May 2012 01:09:39 -0300
>
>  2 + 4 + . . . + 2n.
>  2 + 5 + 8 + . . . + (3n-1).
> Bem eu sei que o primeiro irá dar n(n+1) e o segundo n(3n+1)/2
> O que em si eu não entendi o resultado
> O primeiro eu tentei fazer assim:
> 2+4...+2n
> +
> n+2n+(2n+1),  e fiquei parado nisso e o segunda também, gostaria de uma
> explicação passo-a-passo pois não entendo.

=
Instruções para entrar na lista, sair da lista e usar a lista em
http://www.mat.puc-rio.br/~obmlistas/obm-l.html
=


[obm-l] Re: [obm-l] Re: [obm-l] Re: [obm-l] Re: [obm-l] Dúvida

2011-11-02 Por tôpico Kleber Bastos
Achei que faltava a regra para f(x),mas a menina disse que não. Vou
verificar com a pessoa e retorno.
Obrigado!

Em 02/11/2011 23:08, "Joao Maldonado" escreveu:



Não tem como ser isso não cara
Traduz isso aí que   não dá pra entender

O que poderia ser é
Mostre que  qualquer que seja o número  racional e positivo  a/b com a e  b
inteiros  primos entre si,  é válido   que f(a/b) =  f(1)^(a/b)



Tudo bem,  vamos dizer  que é isto, mas qual a regra para  a função f(x)??

Não são  todas as funções f(x)  que  satisfazem tal  afirmação  (aliás,  a
única   que consegui pensar até agora é  f(x) = a^x

f(x) = x²,  f(4/7) = 16/49 !=   1^(4/7) = 1

[]'s

João



--
Date: Wed, 2 Nov 2011 17:17:59 -0200
Subject: [obm-l] Dúvida
From: klebe...@gmail.com
To: obm-l@mat.puc-rio.br



Olá grupo,
Estou me enrolando nesta prova.

Mostre q ∀ nº a/b>0, MDC(a,b) = 1,
é válido: f(a/b) =...

--
*De:* Kleber Bastos 
*Para:* obm-l@mat.puc-rio.br
*Enviadas:* Quarta-feira, 2 de Novembro de 2011 22:21
*Assunto:* [obm-l] Re: [obm-l] Re: [obm-l] Dúvida


É isso mesmo:
Mostrar que ∀ nº racional a/b>0, M.D.C.(a,b)=1 é válida a sentença:
f(a/b)=f(1)^a/b (...


[obm-l] Re: [obm-l] Re: [obm-l] Re: [obm-l] Dúvida

2011-11-02 Por tôpico Joao Maldonado



Não tem como ser isso não cara
Traduz isso aí que   não dá pra entender



O que poderia ser é
Mostre que  qualquer que seja o número  racional e positivo  a/b com a e  b 
inteiros  primos entre si,  é válido   que f(a/b) =  f(1)^(a/b)    



Tudo bem,  vamos dizer  que é isto, mas qual a regra para  a função f(x)??

Não são  todas as funções f(x)  que  satisfazem tal  afirmação  (aliás,  a 
única   que consegui pensar até agora é  f(x) = a^x

f(x) = x²,  f(4/7) = 16/49 !=   1^(4/7) = 1

[]'s

João






Date: Wed, 2 Nov 2011 17:17:59 -0200
Subject: [obm-l] Dúvida
From: klebe...@gmail.com
To: obm-l@mat.puc-rio.br

Olá grupo,
Estou me enrolando nesta prova.

Mostre q ∀ nº a/b>0, MDC(a,b) = 1, 
é válido: f(a/b) = f(1)^a/b .

-- 
Kleber.



De: Kleber Bastos 
Para: obm-l@mat.puc-rio.br
Enviadas: Quarta-feira, 2 de Novembro de 2011 22:21
Assunto: [obm-l] Re: [obm-l] Re: [obm-l] Dúvida


É isso mesmo:
Mostrar que ∀ nº racional a/b>0, M.D.C.(a,b)=1 é válida a sentença: 
f(a/b)=f(1)^a/b ( f(1) elevado a a/b)


Em 2 de novembro de 2011 20:57, Victor Hugo Rodrigues 
 escreveu:

Como assim? Acho que falta algo aí.
>
>
>Em 2 de novembro de 2011 17:17, Kleber Bastos  escreveu:
>
>
>Olá grupo,
>>Estou me enrolando nesta prova.
>>
>>Mostre q ∀ nº a/b>0, MDC(a,b) = 1, 
>>é válido: f(a/b) = f(1)^a/b
.
>>
>>-- 
>>Kleber.
>>
>


-- 
Kleber B. Bastos

[obm-l] Re: [obm-l] Re: [obm-l] RE: [obm-l] Re: [obm-l] Re: [obm-l] dúvida sobre séries

2011-06-08 Por tôpico Artur Costa Steiner
O critério mais simples para mostrar que a série harmônica diverge talvez
seja o baseado no seguinte teorema:

Se x_n é uma sequência decrescente de reais tal que Soma x_n converge, então
lim n x_ n = 0. (Prove isto)

Se x_n = 1/n, x_n decresce para 0 mas lim n x_n = 1, o que mostra que Soma
x_n diverge. Para infinito, pois os termos são positivos.

Mas talvez não seja uma prova tão elucidativa quanto as outras dadas.

Artur

Artur Costa Steiner
Em 07/06/2011 11:29, "Rodrigo Renji"  escreveu:
>
> Olá!
>
> Então acho bem bacana esse também ( e nem é tão complicado de
> demonstrar, eu acho )
>
>
> Esse critério pode ser usado para estudar a convergência de [ SOMA de
> 1/ k^p  ]  também
>
> pois  [ SOMA de 2^k / 2^(kp)  ]  =  [ SOMA de 2^(k (1-p))  ]
>
> se 1 - p< 0, isto é 1< p a série converge por série geometrica
>
> se 1-p > 0 , 1 > p a série diverge de novo por série geometrica .
>
> =
> Instruções para entrar na lista, sair da lista e usar a lista em
> http://www.mat.puc-rio.br/~obmlistas/obm-l.html
> =


[obm-l] Re: [obm-l] RE: [obm-l] Re: [obm-l] Re: [obm-l] dúvida sobre séries

2011-06-07 Por tôpico Rodrigo Renji
Olá!

Então acho bem bacana esse também ( e nem é tão complicado de
demonstrar, eu acho )


Esse critério pode ser usado para estudar a convergência de [ SOMA de
1/ k^p  ]  também

pois  [ SOMA de 2^k / 2^(kp)  ]  =  [ SOMA de 2^(k (1-p))  ]

se 1 - p< 0, isto é 1< p a série converge por série geometrica

se 1-p > 0 , 1 > p a série diverge de novo por série geometrica .

=
Instruções para entrar na lista, sair da lista e usar a lista em
http://www.mat.puc-rio.br/~obmlistas/obm-l.html
=


[obm-l] RE: [obm-l] Re: [obm-l] Re: [obm-l] dúvida sobre séries

2011-06-07 Por tôpico Luís Lopes

Sauda,c~oes, 

Legal este critério, parece ter sido criado para a série harm. 

E a esse respeito, o autor da pergunta poderia ler também sobre 
a constante de Euler. 

[]'s 
Luís 

> Date: Mon, 6 Jun 2011 23:50:37 -0300
> Subject: [obm-l] Re: [obm-l] Re: [obm-l] dúvida sobre séries
> From: rodrigo.uff.m...@gmail.com
> To: obm-l@mat.puc-rio.br
> 
> Olá!
> 
> Uma outra maneira  ( além da que os colegas enviaram antes), para
> mostrar que a série  não converge, tem um critério de convergência que
> acho legal, Critério de condensação de Cauchy:
> 
> 
>  Se x_k é uma sequência decrescente de termos positivos ( como é o caso de 
> 1/k )
> 
> 
> então a série [ SOMA de x_k]  converge , se e somente se , a série [
> SOMA de 2^k  x_(2^k) ]  converge.
> 
> Aplicando isso para a série do email
> 
> temos com a_k= 1/k
> 
> 
> [ SOMA de 2^k  x_(2^k) ]  =  [ SOMA de 2^k  ,  1/ (2^k)   ] =  [ SOMA 1 ]
> 
> que diverge, pois somando de 1 até n resulta em "n", com n indo pro
> infinito , diverge  : ) Pode não ajudar muito, mas acho esse critério
> legal
> 
> abraço
> 
> =
> Instruções para entrar na lista, sair da lista e usar a lista em
> http://www.mat.puc-rio.br/~obmlistas/obm-l.html
> =
  

[obm-l] Re: [obm-l] Re: [obm-l] Re: [obm-l] Dúvida - OBM Nível Universitário

2011-01-24 Por tôpico Tiago
Seria uma olimpiada mais dificil ainda de ganhar, haha.

2011/1/24 Hugo Fernando Marques Fernandes 

> Oi, Bruna.
>
> Pois é, eu já tinha ouvido dizer isso e queria confirmar.
> É uma pena, mas fazer o que? Regras são regras...
>
> Talvez fosse o caso de criar uma categoria nova pra quem já tem diploma,
> né?
>
> Obrigado pela resposta.
>
> Hugo.
>
> Em 24 de janeiro de 2011 02:20, Bruna Campos escreveu:
>
> PS.: E só até o quarto ano de graduação.
>>
>> Abraços!
>>
>> Em 23/01/11, Bruna Campos escreveu:
>> > Hugo, que eu saiba não pode. Só pode participar quem não tem diploma
>> > de curso superior :(
>> >
>> >
>> >
>> >
>> > Em 20/01/11, Hugo Fernando Marques Fernandes
>> > escreveu:
>> >> Boa noite.
>> >>
>> >> Acabo de ser aprovado para o curso de matemática da UERJ e tenho uma
>> >> dúvida
>> >> em relação à OBM de nível universitário. Sendo esta minha segunda
>> >> graduação,
>> >> ou seja, possuindo um outro diploma de nível superior, ainda assim
>> >> poderei
>> >> participar?
>> >>
>> >> Grato por sua atenção.
>> >>
>> >> Hugo F. M. Fernandes.
>> >>
>> >
>>
>> =
>> Instruções para entrar na lista, sair da lista e usar a lista em
>> http://www.mat.puc-rio.br/~obmlistas/obm-l.html
>> =
>>
>
>


-- 
Tiago J. Fonseca
http://legauss.blogspot.com


[obm-l] Re: [obm-l] RE: [obm-l] Re: [obm-l] dúvida sobre le ma de teoria dos números.

2010-06-05 Por tôpico Bernardo Freitas Paulo da Costa
2010/6/5 Lucas Hagemaister 
>
>  Hum... Entendi. Obrigado!
> O que mais ou menos o lema quer dizer é o seguinte:
> Sempre que termos m|a e n|a, onde mn|a, m e n serão primos entre si.
Tivermos, para não assassinar o português. E não, cuidado com a ordem
das implicações. A e B => C  não quer dizer que A e C => B Veja
bem, podemos ter
20 | 100
5 | 100
20*5 | 100
mas 20 e 5 não são primos entre si.

Muito cuidado com a ordem das coisas... Que nem aquela do "todo corvo
é preto, ora, minha ovelha é preta, logo minha ovelha é um corvo".

> O que eu fiz foi o contrário(ali no caso do 4 e 10):
O que você fez foi usar a conclusão de um teorema sem verificar as
hipóteses... Por exemplo, "todo ser vivo é mortal, logo os
computadores são mortais". Repare que falta o conectivo "ora, os
computadores são seres vivos". (que, por sinal, me parece falsa...)

> Sempre que termos m e n primos entre si, onde m|a e n|a, mn|a.
> Como vimos, no caso do 4 e  10 isso não ocorre.
Justamente, o problema do caso do 4 e do 10 que "faz falhar o
teorema", é que 4 e 10 não são primos entre si, e daí "sobram fatores
primos", que são "contados duas vezes".

Uma sugestão para entender o teorema de Gauss: tente demonstrar sem
essa hipótese. Veja o que falha. Tente corrigir essa primeira
demonstração, sem usar Bézout. A idéia é simplesmente partir da
observação "bom, 3 | x e 2 | x => 6 | x", e tentar generalizar essa
demonstração (que é meio "fácil") para o caso em que 2 e 3 são números
quaisquer. Depois de um certo tempo, acho que vem "naturalmente" a
hipótese certa que falta adicionar ao teorema (que os números são
primos entre si). Mas o grande problema é que agora é praticamente
impossível de generalizar a demonstração que você deu no caso 2,3 =>
6. E essa é uma das partes interessantes (e difíceis) da matemática:
quando você começa a tentar entender mais profundamente alguma coisa,
e depois de um monte de esforço consegue chegar num enunciado que você
acredita suficientemente (ou seja, você esgotou a sua fonte de
contra-exemplos para a situação em questão, e os evitou nas hipóteses,
que capturam a "essência" do que deve fazer funcionar), você descobre
que falta alguma coisa mais forte para demonstrar. Você podia ter
começado com números primos somente (e nesse caso eu acho que tem uma
prova bem simples), mas agora, você precisa usar algo muito mais
sofisticado. E é exatamente entendendo a sofisticação que você
introduz (Bézout é crucial, mesmo, para demonstrar, e você vê
exatamente como ele faz "tudo funcionar"), que você terá entendido o
porquê de duas coisas. A primeira, porque o teorema é verdade, dito
assim. E a segunda, mais importante ainda, é "porque as hipóteses do
teorema são essas aí, e não outras".

E uma outra sugestão. Se você estiver com uma dúvida, escreva sempre
as hipóteses *antes* da conclusão. Escreva [ Se A, B, C e D, Então X
]. Ajuda a evitar erros, e simplifica também bastante na hora de
escrever a contrapositiva, a negação, ... Escrever enunciados de forma
"contorcida" pode ser mais elegante poeticamente, mas é mais ou menos
como usar numerais romanos para fazer multiplicações. Você conseguirá,
mas o algoritmo é muito menos eficiente do que o dos algarismos
arábicos.

Abraços,
--
Bernardo Freitas Paulo da Costa

=
Instruções para entrar na lista, sair da lista e usar a lista em
http://www.mat.puc-rio.br/~obmlistas/obm-l.html
=


[obm-l] Re: [obm-l] RE: [obm-l] Re: [obm-l] dúvida de inter pretação

2009-05-14 Por tôpico Marcelo Costa
*HUMILDEMENTE PEÇO DESCULPAS AOS MEMBROS DA LISTA POR FUGIR AOS PROPÓSITOS
DA MESMA, E AGRADEÇO A BOA VONTADE DO PROF. PALMERIM. DORAVANTE TOMAREI MAIS
CUIDADO AO APRESENTAR PROBLEMAS PARA QUE SEJAM PERTINENTES AOS PROPÓSITOS DA
LISTA. *

2009/5/12 Albert Bouskela 

>  Olá Palmerim,
>
>
>
> Obrigado pela citação!
>
>
>
> Sua resposta está correta e didática. Não obstante, vou pedir-lhe um favor:
> acho que deveríamos parar de elucidar dúvidas tais como a que foi
> apresentada pelo Marcelo. Acredito que seja prudente preservar o propósito
> desta Lista: a discussão de problemas, que sejam, em tese, pelo menos em
> potencial, pertinentes às Olimpíadas de Matemática. Neste sentido, é de todo
> conveniente que a Lista continue voltada para o objetivo fixado pelo Prof.
> Nicolau Saldanha: apoiar a preparação de estudantes para essas Olimpíadas.
> Caso esta Lista passe a ficar poluída por questões do tipo
> “tire-suas-dúvidas-elementares-em-matemática-básica”, certamente vai
> afugentar aqueles participantes que aqui estão para contribuir com o
> propósito original deste fórum. Além disto, estudantes a exemplo do Marcelo,
> podem valer-se de outros fóruns na Internet, bem mais apropriados, para
> sanar as suas dúvidas em Matemática Básica. Podem, outrossim, contar com os
> seus professores.
>
>
>
> *Albert Bouskela*
>
> bousk...@msn.com
>
>
>
> *From:* owner-ob...@mat.puc-rio.br [mailto:owner-ob...@mat.puc-rio.br] *On
> Behalf Of *Palmerim Soares
> *Sent:* Tuesday, May 12, 2009 11:32 AM
> *To:* obm-l@mat.puc-rio.br
> *Subject:* [obm-l] Re: [obm-l] dúvida de interpretação
>
>
>
> Olá Marcelo
>
>
>
> Numa fração os termos são necessariamente números inteiros. Mas uma fração
> pode representar inúmeras coisas: um número, uma divisão, uma RAZÃO etc. A
> Razão na verdade é uma comparação entre duas quantidades, feita por meio da
> divisão entre essas quantidades, as quais podem ser ou não números inteiros.
> Por exemplo, 2/5 é uma fração, e pode estar representando um número, uma
> divisão, e também uma RAZÃO, se estivermos, por exemplo, comparando a
> quantidade de mulheres numa festa em relação à quantidade de convidados.
> Neste caso, a RAZÃO 2/5 quer dizer que de cada 5 convidados, 2 são do sexo
> feminino. Ou em outras palavras, já que 2/5 = 40/100, 40% dos convidados são
> do sexo feminino. Veja que 40/100 é uma fração, mas neste exemplo é também
> uma razão. Então, 40% é uma FRAÇÂO centesimal (denominador igual a 100) e
> também é uma taxa de comparação e, neste sentido, é uma razão. Mas, para os
> puristas, estaria errado dizer que RAIZ(5)/3 é uma fração porque o numerador
> é irracional e não inteiro; pode estar representando uma RAZÃO, mas não é
> uma fração.
>
>
>
> Como diria o mestre Bouskela: Fui claro? :-)
>
>
>
> Abraços
>
> Palmerim
>
>
>
>
>
>
>
> 2009/5/12 Marcelo Costa 
>
> Seguinte:
>
>
>
> Pode-se afirmar que uma porcentagem é uma razão especial, uma razão em que
> o consequente é sempre igual a 100 ?!
>
> Se sim, por ex., 25 % = 25/100 = ¼, não é ?!
>
> Posso ler então, como sendo razão de um para quatro. Está correto ?!
>
> Nesse caso, são 5 partes no total (1 + 4). Onde está a confusão ou o erro
> de interpretação ?!
>
>
>
> Acho que posso afirmar que uma porcentagem é uma fração (e não uma razão)
> em que o denominador é 100
>
>
>
> Por ex,:
>
>
>
> Fração ¼ significa uma parte em quatro.
>
> Razão ¼ significa uma parte para quatro, perfazendo um total de cinco
> partes...
>
>
>
> Favor comentar
>
>
>
>
>
>
>
>
>
>
> --
> "Matemática é o alfabeto com o qual Deus escreveu o Universo"
> Galileu Galilei
>
>
>
>
> --
> Palmerim
>



-- 
"Matemática é o alfabeto com o qual Deus escreveu o Universo"
Galileu Galilei


[obm-l] RE: [obm-l] Re:[obm-l] RE: [obm-l] Dúvida

2004-06-24 Por tôpico Paulo Santa Rita
Oi Claudio,
Se eu nao citei abeliano, foi esquecimento. O Teorema de Cauchy e assim:
"Seja G um grupo FINITO e ABELIANO. Se p e um primo que divide a ordem de G
entao existe um elemento g de G de ordem p".
Esta sua demonstracao ai embaixo e a do Kummer.
Um Abraco
Paulo Santa Rita
5,1356,240604
From: "claudio.buffara" <[EMAIL PROTECTED]>
Reply-To: [EMAIL PROTECTED]
To: "obm-l" <[EMAIL PROTECTED]>
Subject: [obm-l] Re:[obm-l]  RE: [obm-l] Dúvida
Date: Thu, 24 Jun 2004 12:43:59 -0300
Oi, Paulo:
Acho que esta sua demonstracao do teorema de Cauchy soh eh valida se G for 
abeliano, pois no fim, quando voce fala na projecao canonica p: G -> G/H, 
voce estah implicitamente supondo que G/H eh um grupo e, portanto, que H eh 
um subgrupo normal de G. Mas isso soh eh verdade para todo H se G for 
abeliano.

Por outro lado, existe uma demonstracao desse teorema que eh um dos meus 
exemplos favoritos de beleza matematica:

Seja G um grupo e p um primo que divide |G|.
Considere todos os produtos da forma x_1*x_2*...*x_p que sao iguais a "e", 
onde os x_i sao elementos nao necessariamente distintos de G.

Eh facil ver que existem |G|^(p-1) tais produtos pois, escolhendo-se 
livremente os valores de x_1, x_2, ..., x_(p-1), o valor de x_p fica 
unicamente determinado (igual ao inverso de x_1*x_2*...*x_(p-1))

Agora vamos dividir estes |G|^(p-1) produtos em classes de equivalencia de 
forma que dois produtos pertencem a uma mesma classe se e somente se um 
deles for uma permutacao circular do outro. Teremos dois casos a 
considerar:

Caso 1: todos os x_i sao iguais.
Nesse caso, a classe vai conter apenas um produto, pois x_1*x_2*...*x_p = 
a*a*...*a e existe apenas uma permutacao dos x_i.

Caso 2: pelo menos dois dos x_i sao distintos.
Nesse caso, a classe vai conter exatamente p produtos:
x_1*x_2*...*x_(p-1)*x_p;
x_2*x_3*... x_p*x_1;
x_3*x_4*...*x_1*x_2;
...
x_p*x_1*...x_(p-1)*x_(p-1).
Sejam N1 e N2 os numeros de classes de equivalencia de cada tipo.
Entao, teremos que:
numero de produtos =  1*N1 + p*N2 = |G|^(p-1).
Por hipotese, p | |G|^(p-1) e obviamente p | p*N2.
Logo, p | N1.
Alem disso, o produto e*e*...*e obviamente eh do tipo 1, de modo que N1 > 
0.

Ou seja, o numero N1 de produtos da forma a*a*...*a = a^p = e eh um 
multiplo positivo de p.
Em outras palavras, existem pelo menos p-1 elementos em G de ordem p.
Naturalmente, se a eh um tal elemento, entao , o subgrupo ciclico gerado 
por a, terah ordem p.

[]s,
Claudio.
De:[EMAIL PROTECTED]
Para:[EMAIL PROTECTED]
Cópia:
Data:Thu, 24 Jun 2004 14:20:38 +
Assunto:[obm-l] RE: [obm-l] Dúvida

> Ola Eder,
>
> Ok !
>
> Vamos fazer o seguinte. Vou provar um resultado classico que voce podera
> usar na solucao.
>
> TEOREMA DE CAUCHY : Se G e um grupo finito e "p" e um numero primo que
> divide
> a ordem de G entao existe um elemento "g" de G de ordem "p".
>
> PROVA : Vamos usar inducao sobre a ordem de G. Mais especificamente 
vamos
> mostrar que
> ( HIPOTESE DE INDUCAO ) se todos os grupos com ordem menor que G 
satisfazem
> o TEOREMA DE CAUCHY entao G satisfaz o TEOREMA DE CAUCHY.
>
> 1) Se a ordem de G for um numero primo, |G| = p, entao a prova e trivial 
e
> nem precisamos usar a hipotese de inducao, pois "p" sera o unico numero
> primo que pode dividir a ordem de G e se "g"
> for um elemento de G entao, pelo teorema de Lagrange, divide |G|, isto
> e, a ordem de
> "g" e "p". Assim, nao so um, mas todos os elementos de G ( com excecao 
da
> identidade ) tem
> ordem "p"
>
> 2) Se ordem de G nao for um numero primo, seja "p" um numero primo que
> divide a ordem de G.
> Tomando um elemento "g" pertencente a G, "g" diferente de "e", considere 
o
> subgrupo de G : H=. Existem duas possibilidades para H :
>
> PRIMEIRA : H e igual a G. Neste caso, G e ciclico com G=. Seja N=|G| e
> considere o elemento g^(N/p). Claramente que g^(N/p) pertence a G e 
ordem de
> g^(N/p) e "p". Assim,
> G tem um elemento de ordem "p" e acabou.
>
> SEGUNDA : H e diferente de G. Neste caso |H| < |G|.
>
> Se "p" divide |H|, pela HIPOTESE DE INDUCAO, existe "h" pertencente a H 
tal
> que ordem de "h" e "p". Como H e subconjunto de G segue que "h" e tambem
> elmento de G e, portanto, G tem um elemento de ordem "p" e acabou.
>
> Se "p" nao divide |H| ( mas "p" divide |G|, por hipotese ), pelo teorema 
de
> Lagrange |G|=|H|(G:H) teremos que "p" divide (G:H), isto e, "p" divide o
> indice de H em G. Como (G:H) =| G/H | e
> G/H| < |G|, pela HIPOTESE DE INDUCAO, existe um h_ ( h barra ) em G/H de
> ordem "p".
>
> Considere a projecao canonica :
>
> p : G -> G/H
>
> Sabemos que trata-se de um homomorfismo e que em todo homomorfismo a 
ORDEM
> DA IMAGEM DE UM ELEMENTO DIVIDE A ORDEM DO ELEMENTO, isto e, |h_| divide 
|h|
> para algum "h" em G. Como |h_| = p => |h| = kp, para algum k inteiro.
> Considere o elemento h^k. Claramente que h^k pertence a G e | h^k | = p.
> Assim, G tem um elemento de ordem "p".
>
> Vemo que a hipotese de inducao vale ( por vacuidade ) para as ordem 1 e
> tambem para a
> ordem 2. Se

[obm-l] RE: [obm-l] Re:[obm-l] RE: [obm-l] Re:[obm-l] dúvida chara!

2004-05-23 Por tôpico Rogério Moraes de Carvalho
Olá Osvaldo,

Não há a necessidade de formalidades, mesmo porque eu não sou Dr..
Eu gostaria de ressaltar o seguinte comentário que eu coloquei no início dos
meus comentários, caso não tenha ficado claro: "A análise que eu apresento a
seguir corresponde a uma crítica de CARÁTER CONSTRUTIVO com relação à
resolução apresentada pelo Osvaldo. O objetivo desta análise não é depreciar
a resolução do Osvaldo, mas sim de mostrar que é necessário sermos rigorosos
nas resoluções de problemas de Matemática para não chegarmos a resultados 
incorretos. Muitas vezes podemos encontrar uma resposta correta para uma
questão resolvendo-a de maneira errada."

É importante ficar claro que mesmo para o conjunto dos números
inteiros a sua solução está incompleta. Observe que você não verificou se os
valores de "a" e "b" encontrados produzem valores inteiros de "x" e "y".
Neste caso, você não utilizaria a condição a >= b > 0 e encontraria todos os
valores inteiros de "a" e "b" que reproduzem o produto. Veja o exemplo da
questão modificada para constatar que a sua resolução apresentará resultados
inválidos mesmo no conjunto dos números inteiros.

Atenciosamente,

Rogério Moraes de Carvalho
-Original Message-
From: [EMAIL PROTECTED] [mailto:[EMAIL PROTECTED] On
Behalf Of Osvaldo
Sent: domingo, 23 de maio de 2004 17:54
To: obm-l
Subject: [obm-l] Re:[obm-l] RE: [obm-l] Re:[obm-l] dúvida chara!

Desculpe-me se fui parcial Dr., porém equivoquei-me ao 
ler o enunciado da questão. Eu apenas fiz os calculos 
para os números inteiros e não naturais, ou seja, 
inclui algumas possibilidades a mais. 
Obrigado pela observação!





> Olá colegas da lista,
> 
>   Apesar da resolução apresentada pelo Osvaldo 
ter seguido um possível
> raciocínio correto para resolver esta questão, a 
análise dele está
> incompleta porque omite alguns passos muito 
importantes, o que pode nos
> levar a encontrar soluções inválidas. Neste problema 
especificamente, a
> resposta encontrada está correta, porém, se 
modificarmos o valor da
> diferença de quadrados de 27 para outro valor, então 
a resolução dele pode
> nos levar a resultados errados.
> 
>   A análise que eu apresento a seguir 
corresponde a uma crítica de
> caráter construtivo com relação à resolução 
apresentada pelo Osvaldo. O
> objetivo desta análise não é depreciar a resolução 
do Osvaldo, mas sim de
> mostrar que é necessário sermos rigorosos nas 
resoluções de problemas de
> Matemática para não chegarmos a resultados 
incorretos. Muitas vezes podemos
> encontrar uma resposta correta para uma questão 
resolvendo-a de maneira
> errada.
> 
>   Na resolução apresentada abaixo, considere 
que "=>" significa
> "implica" e ">=" significa "maior ou igual a".
> 
> 
> QUESTÃO ORIGINAL:
> 
> "A diferença entre os quadrados de dois números 
naturais é 27. UM dos
> possíveis valores do quadrado da soma desses dois 
números:
> a)529
> b)625
> c)729
> d)841"
> 
> 
> RESOLUÇÃO POSSÍVEL:
> 
> Sejam x e y os dois números naturais, então devemos 
ter:
> x^2 - y^2 = 27 <=> (x + y)(x - y) = 27
> 
> Adotando a = x + y e b = x - y, teremos:
> a.b = 27 (i) (Observe que o produto de a e b é 
positivo)
> Resolvendo o sistema de equações nas variáveis x e 
y, podemos encontrar x e
> y em função de a e b:
> a + b = (x + y) + (x - y) <=> a + b = 2x <=> x = (a 
+ b)/2 (ii)
> a - b = (x + y) - (x - y) <=> a - b = 2y <=> y = (a -
 b)/2 (iii)
> 
> Como x e y são naturais, então x >= 0 e y >= 0. 
Portanto:
> x + y >= 0 + 0 => a >= 0. De acordo com a igualdade 
(i), a não pode ser 0,
> logo a > 0 (iv)
> Como a.b > 0 (i) e a > 0 (iv), então b > 0 (v)
> y >= 0 => -y <= 0 => y >= 0 e 0 >= -y => y >= -y => 
x + y >= x - y =>
> a >= b (vi)
> Por (v) e (vi), concluímos que: a >= b > 0 (vii)
> 
> Sendo assim, devemos encontrar a e b inteiros tais 
que sejam satisfeitas as
> seguintes condições:
> a.b = 27 (ii)
> a >= b > 0 (vii)
> x = (a + b)/2 (ii) seja um número natural.
> y = (a - b)/2 (iii) seja um número natural.
> 
> Analisando os divisores de 27, podemos concluir que 
existem apenas dois
> pares de valores de a e b que satisfazem as 
condições (ii) e (vii):
> (a = 27 e b = 1) ou (a = 9 e b = 3)
> 
> Para a = 27 e b = 1:
> x = (27 + 1)/2 = 14 é um número natural.
> y = (27 - 1)/2 = 13 é um número natural.
> Portanto, x = 14 e y = 13 é uma solução possível.
> 
> Para a = 9 e b = 3:
> x = (9 + 3)/2 = 6 é um número natural.
> y = (9 - 3)/2 = 3 é um número natural.
> Portanto, x = 6 e y = 3 é uma solução possível.
> 
> Possíveis valores para (x

[obm-l] Re:[obm-l] RE: [obm-l] Re:[obm-l] dúvida chara!

2004-05-23 Por tôpico Osvaldo
Desculpe-me se fui parcial Dr., porém equivoquei-me ao 
ler o enunciado da questão. Eu apenas fiz os calculos 
para os números inteiros e não naturais, ou seja, 
inclui algumas possibilidades a mais. 
Obrigado pela observação!





> Olá colegas da lista,
> 
>   Apesar da resolução apresentada pelo Osvaldo 
ter seguido um possível
> raciocínio correto para resolver esta questão, a 
análise dele está
> incompleta porque omite alguns passos muito 
importantes, o que pode nos
> levar a encontrar soluções inválidas. Neste problema 
especificamente, a
> resposta encontrada está correta, porém, se 
modificarmos o valor da
> diferença de quadrados de 27 para outro valor, então 
a resolução dele pode
> nos levar a resultados errados.
> 
>   A análise que eu apresento a seguir 
corresponde a uma crítica de
> caráter construtivo com relação à resolução 
apresentada pelo Osvaldo. O
> objetivo desta análise não é depreciar a resolução 
do Osvaldo, mas sim de
> mostrar que é necessário sermos rigorosos nas 
resoluções de problemas de
> Matemática para não chegarmos a resultados 
incorretos. Muitas vezes podemos
> encontrar uma resposta correta para uma questão 
resolvendo-a de maneira
> errada.
> 
>   Na resolução apresentada abaixo, considere 
que "=>" significa
> "implica" e ">=" significa "maior ou igual a".
> 
> 
> QUESTÃO ORIGINAL:
> 
> "A diferença entre os quadrados de dois números 
naturais é 27. UM dos
> possíveis valores do quadrado da soma desses dois 
números:
> a)529
> b)625
> c)729
> d)841"
> 
> 
> RESOLUÇÃO POSSÍVEL:
> 
> Sejam x e y os dois números naturais, então devemos 
ter:
> x^2 - y^2 = 27 <=> (x + y)(x - y) = 27
> 
> Adotando a = x + y e b = x - y, teremos:
> a.b = 27 (i) (Observe que o produto de a e b é 
positivo)
> Resolvendo o sistema de equações nas variáveis x e 
y, podemos encontrar x e
> y em função de a e b:
> a + b = (x + y) + (x - y) <=> a + b = 2x <=> x = (a 
+ b)/2 (ii)
> a - b = (x + y) - (x - y) <=> a - b = 2y <=> y = (a -
 b)/2 (iii)
> 
> Como x e y são naturais, então x >= 0 e y >= 0. 
Portanto:
> x + y >= 0 + 0 => a >= 0. De acordo com a igualdade 
(i), a não pode ser 0,
> logo a > 0 (iv)
> Como a.b > 0 (i) e a > 0 (iv), então b > 0 (v)
> y >= 0 => -y <= 0 => y >= 0 e 0 >= -y => y >= -y => 
x + y >= x - y =>
> a >= b (vi)
> Por (v) e (vi), concluímos que: a >= b > 0 (vii)
> 
> Sendo assim, devemos encontrar a e b inteiros tais 
que sejam satisfeitas as
> seguintes condições:
> a.b = 27 (ii)
> a >= b > 0 (vii)
> x = (a + b)/2 (ii) seja um número natural.
> y = (a - b)/2 (iii) seja um número natural.
> 
> Analisando os divisores de 27, podemos concluir que 
existem apenas dois
> pares de valores de a e b que satisfazem as 
condições (ii) e (vii):
> (a = 27 e b = 1) ou (a = 9 e b = 3)
> 
> Para a = 27 e b = 1:
> x = (27 + 1)/2 = 14 é um número natural.
> y = (27 - 1)/2 = 13 é um número natural.
> Portanto, x = 14 e y = 13 é uma solução possível.
> 
> Para a = 9 e b = 3:
> x = (9 + 3)/2 = 6 é um número natural.
> y = (9 - 3)/2 = 3 é um número natural.
> Portanto, x = 6 e y = 3 é uma solução possível.
> 
> Possíveis valores para (x + y)^2:
> (x + y)^2 = (14 + 13)^2 = 27^2 = 729
> (x + y)^2 = (6 + 3)^2 = 9^2 = 81
> 
> Resposta: Alternativa c
> 
> 
> Observação: Pode parecer que os passos apresentados 
para deduzir as
> condições são desnecessários, mas são eles que 
garantem a validade das
> soluções encontradas.
> 
> 
> EXPLICAÇÃO DO MOTIVO DA RESOLUÇÃO APRESENTADA PELO 
OSVALDO SER INCOMPLETA:
> 
> Na resolução são apresentados 4 valores possíveis 
para a e b (a,b):
> {(1,27),(3,9),(9,3),(27,1)}. Porém, (1,27) e (3,9) 
não satisfazem a condição
> (vii): a >= b > 0. Portanto, somente os pares (9,3) 
e (27,1) correspondem a
> possíveis valores para a e b, restando apenas 
verificar se eles produzem
> valores naturais para x e y. Logo, na lista de 
valores apresentados para
> (x+y)^2 = a^2, {1, 9, 81, 729}, não poderia aparecer 
os valores 1 = 1^2 e
> nem 9 = 3^2. Além disto, não há garantia de que 81 = 
9^2 e 729 = 27^2
> correspondem a valores de a e b válidos, pois os 
valores de x e y não são
> calculados para verificar se eles são naturais, como 
foi descrito no
> enunciado do problema. Portanto, os valores de a e b 
encontrados poderiam
> não ser válidos. Neste problema específico, os 
valores de a e b encontrados
> são válidos, logo a resposta encontrada está 
correta. A seguir, eu apresento
> uma variação deste problema que mostra de maneira 
concreta que a resolução
> apresentada pelo Osvaldo pode apresentar resultados 
errados. Para se ter uma
> idéia apenas 1 resultado, dos 6 encontrados, é 
correto!
> 
> 
> 
> QUESTÃO MODIFICADA:
> 
> "A diferença entre os quadrados de dois números 
naturais é 68. UM dos
> possíveis valores do quadrado da soma desses dois 
números:
> a)16
> b)289
> c)1156
> d)4624"
> 
> 
> RESOLUÇÃO DO OSVALDO ALTERADA PARA A VERSÃO 
MODIFICADA DA QUESTÃO:
> 
> sejam x e y tais numeros, dai temos que
> x^2-y^2=68
> 
> (x+y)(x-y)=68
> 
> 
> a=x+y

[obm-l] Re:[obm-l] Re: [obm-l] Re:[obm-l] dúvida

2004-04-25 Por tôpico rickufrj
-- Início da mensagem original ---

  De: [EMAIL PROTECTED]
Para: [EMAIL PROTECTED]
  Cc: 
Data: Sun, 25 Apr 2004 10:10:46 -0300
 Assunto: [obm-l] Re: [obm-l] Re:[obm-l] dúvida

> No caso, não entendi o porque do i*(raiz de 1998), 
visto que ao elevarmos ao
> quadrado,  i^2= -1 e a expressão seria -1998
> 
> Creio que apenas (raiz de 1998) seja mais correto.
> 
> Porém a questão não deve ser apenas isto, a e b 
devem pertencer a algum
> conjunto específico como os Inteiros...
> 
> Abraços,
> Rossi
> 
É verdade Rossi , acho que ontem a noite eu estava 
meio 'grog' ... rs
Mas , tenta fazer essa questão por calculo .Tipo , é 
obvio que a diferença mínima é 0 , mas quando vc faz 
isso formalmente , aparece i na raiz . É bem 
estranho , na hora que eu fiz eu não consegui ver o pq 
disso.
Abraço
Luiz H. 

 
__
Acabe com aquelas janelinhas que pulam na sua tela.
AntiPop-up UOL - É grátis!
http://antipopup.uol.com.br/



=
Instruções para entrar na lista, sair da lista e usar a lista em
http://www.mat.puc-rio.br/~nicolau/olimp/obm-l.html
=


[obm-l] Re: [obm-l] Re: [obm-l] Re: [obm-l] Dúvida persistente!!!

2004-04-13 Por tôpico Rafael
Auggy,

Independentemente das contas, a criatividade na construção dos triângulos é
magnífica. Lendo o link, vi que o Cláudio já havia pensado no cálculo da
área por integral e teve uma idéia muito melhor em relação à posição dos
eixos, com origem em B em vez de A.

Enfim, apesar de trabalhoso, é um problema bonito.


Cláudio,

Parabéns por ambas as soluções!


Abraços,

Rafael de A. Sampaio





- Original Message -
From: "Qwert Smith" <[EMAIL PROTECTED]>
To: <[EMAIL PROTECTED]>
Sent: Tuesday, April 13, 2004 5:41 PM
Subject: RE: [obm-l] Re: [obm-l] Re: [obm-l] Dúvida persistente!!!


Sai na geometria mas da umas contas chatas.

na primeira figura:
---
(area em amarelo)  = (area do circulo menor) - 2*(area em verde)

(area em vermelho) = (area do quadrado) -
1/4*{(area do circulo maior) + [(area do quadrado)-(area circulo menor)]} -
(area em verde)
---

na segunda figura:
---
(area em laranja) = (setor circular PBQ) - 2*(triangulo PBO - area em azul)
(area em verde) = (setor circular POQ) - (area em laranja)
--

Como todos os lados do triangulo sao conhecidos (em funcao do lado do
quadrado).  Agulos e areas sao questao de braco.

A descricao da figura 2 vc encontra no link

http://www.mat.puc-rio.br/~nicolau/olimp/obm-l.200310/msg00574.html

em uma menssagem do grande Claudio BUffara que ainda teve paciencia
montruosa de me explicar em off o problema.

Em meu email anterior eu tinha feito confusao e atribuido a mensagem do link
acima a outro fera, o Paulo Santa Rita que mandou uma mensagem sobre "lua
algebrica", que tb vale a pena conferir.
E' muito genio pra keep track.

Valeu Super Buffara!

[]s,
Auggy



=
Instruções para entrar na lista, sair da lista e usar a lista em
http://www.mat.puc-rio.br/~nicolau/olimp/obm-l.html
=


[obm-l] Re: [obm-l] Re: [obm-l] Re: [obm-l] dúvida

2004-02-28 Por tôpico Rafael
Tarcio,

O enunciado não dá margem a interpretar que se tratem de grupos formados
para os quais a ordem de escolha importa. ("Dispomos de 10 livros diferentes
e queremos organizar grupos de três livros. O número de grupos diferentes
que podemos formar é igual a...?", observe que os "grupos" não foram
definidos, não se sabe se são grupos de livros que possuem a mesma capa, que
fazem parte de uma mesma coleção, absolutamente nada. Só se sabe que são
livros diferentes e que os grupos possuem três deles.) O Nicolau, aliás, deu
um exemplo bastante consistente sobre isso. Mas não é a primeira vez que
vejo enunciados interpretados de forma incorreta por quem resolveu. Certa
vez, aconteceu comigo quando estava estudando permutações caóticas, o que
realmente foi um caos, pois era um dos primeiros exercícios que eu resolvia.
A única sugestão é procurar bons livros para começar, a coleção do Iezzi
possui um livro excelente de Combinatória (volume 5), e depois que você
tiver os conceitos bem claros já saberá quando algo é incoerente.

Aproveitando a oportunidade, alguns dias atrás, você enviou novamente aquele
problema de capital que decuplicou, e nenhuma das alternativas está correta
para o enunciado dado. Por curiosidade, calculei aplicando juros simples e
juros compostos, comparando com a alternativa correta que você havia dito
(12/7% a.m.). Veja:

Seja C o capital envolvido, a juros simples, teremos:

10C = C(1+i*7*12) ==> i = 0,10714285714...

A juros compostos, teremos:

10C = C(1+i)^(7*12) ==> i = 0,02779088522...

Porém, 12/7% = 0,01714285714...

E, ainda assim, há uma diferença de 1% (aprox.) entre a resposta dada como
correta e a taxa de juros, considerando que fossem compostos, e não simples
como dito no enunciado.

Viu só?


Abraços,

Rafael de A. Sampaio




- Original Message -
From: "Tarcio Santiago" <[EMAIL PROTECTED]>
To: <[EMAIL PROTECTED]>
Sent: Saturday, February 28, 2004 8:16 PM
Subject: [obm-l] Re: [obm-l] Re: [obm-l] dúvida


AMIGO RAFAEL OBRIGADO POR SUA AJUDA. A RESPOSTA É 720, MAS EU ACHAVA QUE O
CERTO ERA 120, POIS O LIVRO  O TRIO DE LIVROS A,Be C é igual a B,CeA . estou
errado?
a questão dar 120 ou 720 ?
estou encucado!!?

=
Instruções para entrar na lista, sair da lista e usar a lista em
http://www.mat.puc-rio.br/~nicolau/olimp/obm-l.html
=


[obm-l] RE: [obm-l] Re: [obm-l] RE: [obm-l] Dúvida Simples!!!

2004-01-13 Por tôpico Paulo Santa Rita
Ola Pessoal,

Eu cometi um erro de digitacao e uma de minhas respostas e, em funcao disso, 
vou falar um pouco mais sobre estas coisas, bastante conhecidas :

Se Y=F(X) e uma funcao e queremos mostrar que ela e injetiva, nos fazemos :
x1 # x2  =>  F(x1) # F(x2)   - aqui, o simbolo "#", siginifica : e diferente 
de.

Pode-se provar isso negando a tese, o que da : F(x1)=F(x2)  =>  x1=x2.
No caso da funcao Y=2x-5, nos podiamos por :
x1=x2  <=>  2*x1=2*x2  <=> 2*x1 - 5 = 2*x2 - 5 <=> F(x1)=F(x2)
A implicacao X1=x2 => F(x1)=F(x2) e desnecessaria, pois, sendo F uma funcao, 
um elemento qualquer do dominio nao pode ter mais de uma imagem no 
contra-dominio. Todavia, quando nos escrevemos, pensamos em que esta lendo e 
pode ser que a dupla implicacao torne a sequencia de raciocinios mais clara, 
sobretudo pra principiantes e foi justamente o que eu queria fazer, mas 
coloquei => onde deveria ter colocado <=>.

Assim, e certo fazer :
x1=x2  <=>  2*x1=2*x2  <=> 2*x1 - 5 = 2*x2 - 5 <=> F(x1)=F(x2)
Como seria certo fazer :
F(x1)=F(x2) => 2*X1-5=2*x2 - 5 => 2*x1=2*x2 => x1 = x2
Quando nos usamos isso, estamos, em verdade, usando o fato :

A => B  <=> ~B => ~A
Ou seja : Provar : x1 # X2 => F(x1) # F(x2)  - Funcao injetiva
E equivalente a provar : F(x1) = F(x2) => X1 = X2.
Um Abraco a Todos
Paulo Santa Rita
3,0956,130104
From: "Paulo Santa Rita" <[EMAIL PROTECTED]>
Reply-To: [EMAIL PROTECTED]
To: [EMAIL PROTECTED]
Subject: [obm-l] Re: [obm-l] RE: [obm-l] Dúvida Simples!!!
Date: Tue, 13 Jan 2004 11:16:36 +
MIME-Version: 1.0
X-Originating-IP: [200.142.58.18]
X-Originating-Email: [EMAIL PROTECTED]
X-Sender: [EMAIL PROTECTED]
Received: from mc1-f30.hotmail.com ([64.4.50.37]) by mc1-s2.hotmail.com 
with Microsoft SMTPSVC(5.0.2195.6824); Tue, 13 Jan 2004 03:25:55 -0800
Received: from saci.mat.puc-rio.br ([139.82.27.51]) by mc1-f30.hotmail.com 
with Microsoft SMTPSVC(5.0.2195.6824); Tue, 13 Jan 2004 03:25:01 -0800
Received: from saci.mat.puc-rio.br (localhost [127.0.0.1])by 
saci.mat.puc-rio.br (8.12.8/8.12.8) with ESMTP id i0DBG7xF025951for 
<[EMAIL PROTECTED]>; Tue, 13 Jan 2004 09:16:07 -0200
Received: (from [EMAIL PROTECTED])by saci.mat.puc-rio.br 
(8.12.8/8.12.8/Submit) id i0DBG7Jp025949for obm-l-MTTP; Tue, 13 Jan 2004 
09:16:07 -0200
Received: from hotmail.com (sea2-f29.sea2.hotmail.com [207.68.165.29])by 
saci.mat.puc-rio.br (8.12.8/8.12.8) with ESMTP id i0DBG5xF025946for 
<[EMAIL PROTECTED]>; Tue, 13 Jan 2004 09:16:06 -0200
Received: from mail pickup service by hotmail.com with Microsoft SMTPSVC; 
Tue, 13 Jan 2004 03:16:36 -0800
Received: from 200.142.58.18 by sea2fd.sea2.hotmail.msn.com with HTTP;Tue, 
13 Jan 2004 11:16:36 GMT
X-Message-Info: o8IIVuzO8A0xt1jbTtkAABvddtGJF13ACLo6v1RyJA0=
Message-ID: <[EMAIL PROTECTED]>
X-OriginalArrivalTime: 13 Jan 2004 11:16:36.0631 (UTC) 
FILETIME=[B5614A70:01C3D9C6]
Precedence: bulk
Return-Path: [EMAIL PROTECTED]

Ola Fabio e demais colegas
desta lista ... OBM-L,
Nao precisa inverter, basta usar o sinal <=> no lugar de =>, que e o que eu 
queria fazer.

From: Fábio Dias Moreira <[EMAIL PROTECTED]>
Reply-To: [EMAIL PROTECTED]
> 1) f(x)=2x-5 ( Dominio : R, Contra-Dominio : R )
>
> Suponha que x1=x2. Entao :
> 2*x1 = 2*x2   =>   2*x1 - 5 = 2*x2 - 5   =>   f(x1)=f(x2)
> Portanto : x1=x2 => f(x1)=f(x2) => funcao injetiva.
> [...]
A implicação x=y => f(x)=f(y) vale para qualquer função trivialmente.  A
implicação que prova a injetividade é f(x)=f(y) => x=y (ou, naturalmente, 
a
sua contrapositiva).

De qualquer forma, basta inverter a cadeia de implicações acima.

[]s,

- --
Fábio "ctg \pi" Dias Moreira
-BEGIN PGP SIGNATURE-
Version: GnuPG v1.2.3 (GNU/Linux)
iD8DBQFAAvaualOQFrvzGQoRAudzAJwKZmwKUbGWJepRhwJbXgzpRl+lhQCcDHUe
lmPqaHD0ss5v6t63HcZqlVE=
=aZJ1
-END PGP SIGNATURE-
=
Instruções para entrar na lista, sair da lista e usar a lista em
http://www.mat.puc-rio.br/~nicolau/olimp/obm-l.html
=
_
MSN Hotmail, o maior webmail do Brasil.  http://www.hotmail.com
=
Instruções para entrar na lista, sair da lista e usar a lista em
http://www.mat.puc-rio.br/~nicolau/olimp/obm-l.html
=
_
MSN Messenger: converse com os seus amigos online.  
http://messenger.msn.com.br

=
Instruções para entrar na lista, sair da lista e usar a lista em
http://www.mat.puc-rio.br/~nicolau/olimp/obm-l.html
=


[obm-l] Re: [obm-l] Re:[obm-l] RE: [obm-l] Dúvida (urgente)

2003-11-26 Por tôpico Rogerio Ponce
Olá Osvaldo,
para encontrar a interseção de f() com uma reta , você está precisando 
calcular sucessivas interseções da mesma função f() com uma 
circunferência...
É original, mas acho que não faz muito sentido, certo ?

Abraços,
Rogério.




From: "Osvaldo" <[EMAIL PROTECTED]>

Está certo, a circunferencia tem raio f(X0).

Estou tentando desenvolvendo um metodo numerico para
calcular as raizes de uma funcao continua de modo que
necessite de MUITO MENOS interacoes com relacao as
necessarias usando o famoso metodo de newton (usando
derivacoes). Para isto pego um pto. da funcao e traco um
circunferencia de raio f(Xo), tocando no eixo dos X.
Encontrando as coodenadas de todas as  interseccoes de f
com a circunferencias pego a de abssissa menor e traco
outra circunferencia, agora com centro em (x1,f(x1))
tocando o eixo dos x novamente e assim sucessivamente,
porem tenho que encontrar x1=F(xo,f(xo)), onde F é a
funcao a ser determinada, e x1 NAO pode estar em funcao
de f(x1).
Fiz inumeros testes para este processo, funcionou!!!,
mais nao consigo provar analiticamente.
Na sua resposta

> f(x1)= f(x0) + sqrt(f(x0)^2 - (x1-x0)^2) ou
> f(x1)= f(x0) - sqrt(f(x0)^2 - (x1-x0)^2
x1 esta em funcao de f(x1), que nao é a funcao que
queria.
Agradeco sua ajuda.









> Osvaldo,
>
> Nao sei se entendi direito, me corriga se eu estiver er
rado.
>
> Considere dois pontos P1 e P2 tais que:
>
> P1: (X0,F(X0))  -
  Centro da Circunferencia (Why ??? Faca um desenho)
> P2: (X1,F(X1))  -
  Ponto de intersecao de f com a circunferencia.
>
> Note, a circunferencia tem que ter centro P1 e o raio d
ela tem que ser f(X0)
> segundo a descricao do problema
(Faca um desenho). Alem disso, se f
> intersecta a circunferencia em P2, entao temos que ter:
>
> |P1-P2| = Raio da circunferencia = f(X0)
>
> (x1-x0)^2 + (f(x1)-f(x0))^2 = [f(x0)] ^2
>
> [f(x1) - f(x0)]^2 = f(x0)^2 - (x1-x0)^2
>
> f(x1)= f(x0) + sqrt(f(x0)^2 - (x1-x0)^2) ou
> f(x1)= f(x0) - sqrt(f(x0)^2 - (x1-x0)^2).
>
>
> Regards,
>
> Leandro
> Los Angeles, CA.
>
>
> -Original Message-
> From: [EMAIL PROTECTED] [mailto:owner-obm-
[EMAIL PROTECTED] On
> Behalf Of Osvaldo
> Sent: Wednesday, November 26, 2003 2:06 PM
> To: lista de discussao de matematica
> Subject: [obm-l] Dúvida (urgente)
>
> Olá pessoal, tenho um problema que tenho tentado
> solucionar mas tá dificil. Muitos tem me dito que é
> impossível, mas eu insisto.
>
> O problema é o seguinte:
>
> "Seja f uma função contínua em seu domínio. Sabe-
se que
> ela passa pelo centro de uma circunferência que é
> tangente ao eixo dos X na abscissa Xo. A função não é
> necessariamente bijetora e seja X1 a abscissa de uma da
s
> intersecções de f com a circunferencia em questão.
>
> O problema é determinar f(X1) EM TERMOS DE Xo E/OU F
> (Xo)." - Paradigma de Labaki-Osvaldo
>
>
>
>
> Eu substitui X1 na equação da circunferência e dirivei-
a
> com relação a X1 duas vezes consecutivas obtendo, assim
,
> uma expressão para a derivada segunda em X1 da função
> dada em termos de Xo e de f
(Xo). Daí teria que encontrar
> as raízes desta equação diferenciavél, mais não consegu
i
> encontrar f(0) nem f'(0), o que complica mais.
>
>
>
> Atenciosamente,
>
> Osvaldo Mello Sponquiado FEIS - UNESP
> Usuário em GNU/Linux
> Futuro Engenheiro Eletricista
>
>
>
>
> ___
___
> Acabe com aquelas janelinhas que pulam na sua tela.
> AntiPop-up UOL - É grátis!
> http://antipopup.uol.com.br/
>
>
>
> ===
==
> Instruções para entrar na lista, sair da lista e usar a
 lista em
> http://www.mat.puc-rio.br/~nicolau/olimp/obm-l.html
> ===
==
>
> ===
==
> Instruções para entrar na lista, sair da lista e usar a
 lista em
> http://www.mat.puc-rio.br/~nicolau/olimp/obm-l.html
> ===
==
>
Atenciosamente,

Osvaldo Mello Sponquiado FEIS - UNESP
Usuário em GNU/Linux




__
Acabe com aquelas janelinhas que pulam na sua tela.
AntiPop-up UOL - É grátis!
http://antipopup.uol.com.br/


=
Instruções para entrar na lista, sair da lista e usar a lista em
http://www.mat.puc-rio.br/~nicolau/olimp/obm-l.html
=
_
MSN Messenger: converse com os seus amigos online.  
http://messenger.msn.com.br

=
Instruções para entrar na lista, sair da lista e usar a lista em
http://www.mat.puc-rio.br/~nicolau/olimp/obm-l.html
=


[obm-l] RE: [obm-l] Re: [obm-l] Re: [obm-l] Dúvida de vestibular

2003-02-27 Por tôpico João Gilberto Ponciano Pereira
Este é um problema interessante! Mas acho que faltou dizer que as cidades em
questão fazem parte do mesmo país, ou seja, a cidade A pertence a um país C
se existe pelo menos uma estrada que vá de A para alguma cidade pertencente
a C.

Acho que a solução é mais ou menos assim:

A "pegadinha" é provar que se existe um caminho que vai de Cn para Cm, então
existe o caminho de volta de Cm para Cn. Prova: seja Pi o "peso" do número
de estradas da cidade Ci, que é o número de estradas que entram mais o
número de estradas que saem. Supondo que o viajante pega o carro de Cm e vai
para Cn passando por algumas cidades. Como ele saiu de Cm, o peso Pm será 1.
Para as outras cidades, o peso é 2i, pois ele chegou e saiu. Para Cn, o peso
é 2i+1, pois ele chegou.

Veja que as cidades devem ter Pi par, pois o número de estradas que chegam é
o mesmo das que saem. Logo, o ciclo de volta sempre terminará em Cm.

Suponha agora um país com 2 cidades A e B. Se A chega a B, temos que B chega
a A, e o número de mapas é igual. Acrescentando uma cidade C no país, ela
será ligada em A, sem perda de generalidade. Ora, se A chega em C e B chega
em A, então B chega em C e, pela volta, C chega em B. Logo, por indução, vc
consegue chegar em qualquer cidade de um determinado país com N cidades, o
número de mapas possíveis é N-1.

-Original Message-
From: Nicolau C. Saldanha [mailto:[EMAIL PROTECTED]
Sent: Thursday, February 27, 2003 1:41 PM
To: [EMAIL PROTECTED]
Subject: [obm-l] Re: [obm-l] Re: [obm-l] Dúvida de vestibular


Eu [ainda] não sei resolver o problema do Okakome mas...

On Wed, Feb 26, 2003 at 04:39:33PM -0300, Domingos Jr. wrote:
> > Oi Pessoal,
> >   Estava estudando análise combinatória por uma
> > apostila de um curso pré-vestibular, e encontrei o
> > seguinte problema, que achei interessante, mas minha
> > solução foi muito longa, e não sei se está certa,
> > porque tinha muitos casos. Se estivesse num
> > vestibular, o que faria?
> >   Num país, as estradas ligam duas cidades e são de
> > mão única (pode haver mais de uma estrada entre duas
> > cidades). O número de estradas que partem de cada
> > cidade é igual ao número de estradas que chegam nessa
> > cidade. Um mapa da cidade C é um conjunto de rotas
> > que: 1) levam C a cada uma das outras cidades do país,
> > sem passar por uma cidade mais de uma vez. 2) Se uma
> > rota parte de C a D passando por E, então a rota que
> > vai de C a E coincide com o começo da rota de C a D.
> > Prove que o número de mapas da cidade C é igual ao
> > número de mapas de qualquer outra cidade.
> 
> Acho que esse enunciado não está completo:
> - se existe uma cidade com nenhuma estrada partindo ou chegando possui um
> número de rotas 0 e não vai ser igual as demais, até aí é um caso idiota
que
> pode ser excluído do problema.

Neste caso não existe nenhum mapa pois é impossível ligar C a X,
a cidade sem estradas. O problema fica correto pois o número
de mapas é sempre 0.

> - se existem conjuntos de cidades "disjuntos" ou seja cidades de um
conjunto
> A não possuem rota nenhuma para as cidades do conjunto B e vice-versa:
> 
> C1 <---> C2 ( 1 estrada de C1 -> C2 e outra de C2 -> C1  )
> C3 <==> C4 ( 2 estradas de C3 -> C4 e duas de C4 -> C3 )
> 
> neste caso temos que o número de rotas de N(C1) = N(C2) = 1, mas N(C3) =
> N(C4) = 2.

Também neste caso o número de mapas é sempre 0.

> além disso, há um trecho que eu considero confuso:
> 
> "2) Se uma rota parte de C a D passando por E, então a rota que vai de C a
E
> coincide com o começo da rota de C a D"
> nessa situação parece que a rota de C a E deve ser única, mas podem haver
> outras rotas de C até E sem que uma cidade seja visitada mais de uma
vez...
> mesmo que sempre fossem escolhidos os caminhos que passem por menos
cidades
> isso poderia ocorrer já que é permitido haver mais de uma estrada ligando
> duas cidades.

Acho que ajuda considerar o caso em que todas as estradas são de mão dupla:
neste caso um mapa nada mais é do que uma árvore maximal e o problema
fica trivial.

[]s, N.
=
Instruções para entrar na lista, sair da lista e usar a lista em
http://www.mat.puc-rio.br/~nicolau/olimp/obm-l.html
O administrador desta lista é <[EMAIL PROTECTED]>
=
=
Instruções para entrar na lista, sair da lista e usar a lista em
http://www.mat.puc-rio.br/~nicolau/olimp/obm-l.html
O administrador desta lista é <[EMAIL PROTECTED]>
=


[obm-l] Re: [obm-l] Re: [obm-l] Re: [obm-l] Dúvida de vestibular

2003-02-27 Por tôpico Nicolau C. Saldanha
On Thu, Feb 27, 2003 at 01:40:56PM -0300, Nicolau C. Saldanha wrote:
> Eu [ainda] não sei resolver o problema do Okakome mas...

Aliás, Okakamo. Desculpe. De que origem é este nome?
Procurando no Google encontrei um Okakamo Matsubachi 
mencionado na Eureka 14 e um Kokobongo em
http://cs.tklan.com.br:8001/stats/player_a29rb2Jvbmdv.html
que parece ser o apelido de um jogador em algum tipo de jogo.
Também há um CD Kokobongo mencionado em
http://sites.uol.com.br/marcomarrero/planeta/planeta0709/pagina1.htm
e um Kokobongo Bar em Balneário Gaivota:
http://www.jmnet.com.br/colunistas/claudete_matos.htm
O nome Kokobongo também aparece em uma página em japonês:
http://hamq.jp/stdB.cfm?i=DREAMNIGHT&pn=13

[]s, N.
=
Instruções para entrar na lista, sair da lista e usar a lista em
http://www.mat.puc-rio.br/~nicolau/olimp/obm-l.html
O administrador desta lista é <[EMAIL PROTECTED]>
=


[obm-l] Re: [obm-l] Re: [obm-l] Re: [obm-l] Re: [obm-l] Re: [obm-l] dúvida

2002-10-01 Por tôpico Douglas Carvalho

Pois vc representa os numeros com as ordens
crescendo da dir para esq.
Por exemplo:

512 (decimal)

é 2 * 10^0 (unidade)
+
   1 * 10^1 (dezena)
+
   5 * 10^2 (centena)

assim como

10100 (binário)
é:

0 * 2^0 +
0 * 2^1 +
1 * 2^2 +
0 * 2^3 +
1 * 2^4 =
 20 em decimal.

Assim, no problema proposto, o 2^0 seria
a primeira questão, 2^1 seria a 2ª e assim
por diante.

Amplexos,

Douglas

-Mensagem Original-
De: "Henrique Branco" <[EMAIL PROTECTED]>
Para: <[EMAIL PROTECTED]>
Enviada em: Terça-feira, 1 de Outubro de 2002 19:35
Assunto: [obm-l] Re: [obm-l] Re: [obm-l] Re: [obm-l] Re: [obm-l] dúvida


> Eder, desculpe, acho que minha pergunta que foi mal colocada.
> A questão é justamente "por que contar da esquerda para a direita"?
> Já que a 610 na base 2 é 1001100010, por que não contar como se ele
tivesse
> acertado a primeira, a quarta, quinta...? (apesar de que isso não
satisfaria
> a questão, pois geraria uma resposta errada)
>
> Grato,
> Henrique.
>
> - Original Message -
> From: "Eder" <[EMAIL PROTECTED]>
> To: <[EMAIL PROTECTED]>
> Sent: Tuesday, October 01, 2002 1:54 PM
> Subject: [obm-l] Re: [obm-l] Re: [obm-l] Re: [obm-l] dúvida
>
>
> > 0010 0100 0001,mas não aparecem 0110 ou 0010 novamente
> > O que isso tem de especial?
> >
> > Na hora de somarmos as pontuações,os 1's aparecerão  na ordem das
> perguntas
> > respondidas corretamente!De fato,basta lembrar como se soma números
> > binários,como teremos sempre 1 "em cima de zero" ou "zero em cima de
> > zero"
> >
> > Pois é,basta representar o resultado na base 2 e contar da direita para
> > esquerda,essa é a ordem das perguntas.Daí ele ter respondido as
perguntas
> > 2,6,7 e 10.Note que a representação na base 2 não compromete,ou melhor,
> ele
> > está por trás de tudo!Caso a pergunta 1 tivesse sido respondida
> corretamente
> > o candidato ganharia 1*2º=1 ponto,conforme expresso no enunciado.Espero
> ter
> > ajudado.
>
>
> =
> Instruções para entrar na lista, sair da lista e usar a lista em
> http://www.mat.puc-rio.br/~nicolau/olimp/obm-l.html
> O administrador desta lista é <[EMAIL PROTECTED]>
> =
>

=
Instruções para entrar na lista, sair da lista e usar a lista em
http://www.mat.puc-rio.br/~nicolau/olimp/obm-l.html
O administrador desta lista é <[EMAIL PROTECTED]>
=



[obm-l] Re: [obm-l] Re: [obm-l] Re: [obm-l] Re: [obm-l] dúvida

2002-10-01 Por tôpico Henrique Branco

Eder, desculpe, acho que minha pergunta que foi mal colocada.
A questão é justamente "por que contar da esquerda para a direita"?
Já que a 610 na base 2 é 1001100010, por que não contar como se ele tivesse
acertado a primeira, a quarta, quinta...? (apesar de que isso não satisfaria
a questão, pois geraria uma resposta errada)

Grato,
Henrique.

- Original Message -
From: "Eder" <[EMAIL PROTECTED]>
To: <[EMAIL PROTECTED]>
Sent: Tuesday, October 01, 2002 1:54 PM
Subject: [obm-l] Re: [obm-l] Re: [obm-l] Re: [obm-l] dúvida


> 0010 0100 0001,mas não aparecem 0110 ou 0010 novamente
> O que isso tem de especial?
>
> Na hora de somarmos as pontuações,os 1's aparecerão  na ordem das
perguntas
> respondidas corretamente!De fato,basta lembrar como se soma números
> binários,como teremos sempre 1 "em cima de zero" ou "zero em cima de
> zero"
>
> Pois é,basta representar o resultado na base 2 e contar da direita para
> esquerda,essa é a ordem das perguntas.Daí ele ter respondido as perguntas
> 2,6,7 e 10.Note que a representação na base 2 não compromete,ou melhor,
ele
> está por trás de tudo!Caso a pergunta 1 tivesse sido respondida
corretamente
> o candidato ganharia 1*2º=1 ponto,conforme expresso no enunciado.Espero
ter
> ajudado.


=
Instruções para entrar na lista, sair da lista e usar a lista em
http://www.mat.puc-rio.br/~nicolau/olimp/obm-l.html
O administrador desta lista é <[EMAIL PROTECTED]>
=



[obm-l] Re: [obm-l] Re: [obm-l] Re:[obm-l] dúvida??

2002-10-01 Por tôpico Paulo Santa Rita

Ola Leonardo e demais
colegas desta lista ... OBM-L,

Oi Leonardo, tudo legal ?

Voce esta certo. O PRIMEIRO PULO ocorre quando ela BATE NO CHAO PELA 
PRIMEIRA VEZ, quando, portanto, fora largado da ALTURA INICIAL de 12 metros. 
Segue que a altura que ela atinge neste primeiro pulo e de
12*(2/3).

Esse ultimo valor que encontramos sera a base para se calcular qual a altura 
que ela atinge no segundo pulo, que sera : 12*[(2/3)^2]. E assim 
sucessivamente. No terceiro pulo ela atinge a altura de 12*[(2/3)^3], isto 
e,  32/9

Claramente que no N-esimo pulo ela atinge a altura de 12[(2/3)^N]

Quando ela bate no chao pela N-esima vez e porque ela ja deu N-1 pulos. em 
cada pulo e necessario considerar a SUBIDA E DESCIDA, logo :

Sn = 12 + 2*12*(2/3) + 2*12*(2/3)^2 + ... + 2*12*(2/3)^(N-1)

e o espaco que ela EFETIVAMENTE JA PERCORREU no momento em que toca no solo 
pela N-esima vez ! Em particular, ao tocar no solo pela terceira vez, tera 
efetivamente percorrido :

S3 = 12 + 2*12(2/3) + 2*12*(2/3)^2 = 116/3

Se nos quisessemos encontrar quanto ela teria percorrido ate parar bastaria 
somar a PG infinita. Isto daria :

S = 12 + PG infinita = 12 + [16/(1 - 2/3)] = 12 + 48 = 60

Isto tudo e muito simples, cai em vestibulares e constitui um conhecimento 
basico : Numa PG de termos a1, a2, ..., an se a razao "q" e tal que 
modulo(q) < 1 entao existe :

Lim Sn ( N indo pro infinito ) e Lim Sn = a1 /(1-q).

Em verdade, o passo crucial aqui e ver que se :

Sn = a1 + a1*q + a1*(q)^2 + ... + a1*[q^(N-1)]

Entao

(1-q)*Sn = a1*(1 - q^N) e como q^N -> 0 quando N->+INF segue que
(1-q)*Sn = a1.

Vemos que o polinomio P=f(q)=1-q foi muito util, pois tornou a consideracao 
do limite facil. Agora,

Observando que os expoente de "q" numa PG tradicional constituem uma PA 
ordinaria, isto e, uma PA de ordem 1, podemos imaginar uma "PG" de ordem P 
como aquela "serie geometrica" em que os expoentes de "q" constituem uma PA 
de ordem P. Assim,

a1, a1*q, a1*(q^3), a1*(q^6), ... ,a1*^[q^(N*(N-1)/2)], ...

Claramente que se modulo(q) < 1 a serie :

Sn= a1 + a1*q + a1*(q^3)+ a1*(q^6)+ ...+ a1*^[q^(N*(N-1)/2)]+ ...

converge. Sera que, assim como na serie geometrica, existe um polinomio 
p=f(q) tal que :

f(q)*Sn permita calcular com facilidade o LIM Sn ? Se sim, qual a cara ( a 
forma ) deste polinomio ?

Um Abraco
Paulo Santa Rita
3,1746,011002

>From: "leonardo mattos" <[EMAIL PROTECTED]>
>Reply-To: [EMAIL PROTECTED]
>To: [EMAIL PROTECTED]
>Subject: [obm-l] Re: [obm-l] Re:[obm-l] dúvida??
>Date: Tue, 01 Oct 2002 00:51:25 +
>
>  Ola,
>  Quanto a letra a).Pq nao seria 12*(2/3)^3?!Considerando que no 1ºpulo ela 
>alcança 12*2/3,2ºpulo 12*(2/3)^2 e no 3º12*(2/3)^3...
>   Um abraço,Leonardo
>
>
>>From: "Wagner" <[EMAIL PROTECTED]>
>>Reply-To: [EMAIL PROTECTED]
>>To: <[EMAIL PROTECTED]>
>>Subject: [obm-l] Re:[obm-l] dúvida?? Date: Mon, 30 Sep 2002 20:31:27 -0300
>>
>>Oi pessoal !
>>
>>
>>Mário wrote:
>>
>>Amigos de lista, peço ajuda:
>>
>>
>>"Uma bola pula cada vez que bate no chão 2/3 da altura de onde caiu.
>>Deixando-a cair da altura de 12 metros, pergunta-se:
>>a) qual será a altura do terceiro pulo?
>>b) Quanto percorreu ao bater no chão pela terceira vez?
>>
>>
>>  As alturas máximas a cada pulo estão em progressão geométrica de razão 
>>2/3 e termo inicial 12, assim como a distância percorrida a cada pulo 
>>também forma uma PG de razão 2/3 e termo inicial 24. Logo:
>>
>>a) x = 12.(2/3)^2 = 16/3 metros
>>b) Considerando que a pergunta seja quanto ela percorreu desde que é 
>>abandonada da altura inicial :
>>y = 24.((2/3)^3 - 1)/(2/3 -1) = 24.(-19/27)/(-1/3) = 24.3.19/27 = 152/3 
>>metros
>
>
>
>
>_
>Converse com seus amigos online, faça o download grátis do MSN Messenger: 
>http://messenger.msn.com.br
>
>=
>Instruções para entrar na lista, sair da lista e usar a lista em
>http://www.mat.puc-rio.br/~nicolau/olimp/obm-l.html
>O administrador desta lista é <[EMAIL PROTECTED]>
>=




_
MSN Photos é a maneira mais fácil e prática de editar e compartilhar sua 
fotos: http://photos.msn.com.br

=
Instruções para entrar na lista, sair da lista e usar a lista em
http://www.mat.puc-rio.br/~nicolau/olimp/obm-l.html
O administrador desta lista é <[EMAIL PROTECTED]>
=



[obm-l] Re: [obm-l] Re: [obm-l] Re: [obm-l] dúvida

2002-10-01 Por tôpico Eder

Seguindo o racionínio do Villard,veja que se representarmos na base 2 o
valor dos pontos ganhos em cada pergunta ,teremos sempre representações
diferentes.Teremos sempre um "1" e alguns zeros,ou apenas zeros (no caso de
repostas erradas).Nunca teremos duas pontuações com 1's na mesma
posição.Exemplos hipotéticos de pontuações:


0010 0100 0001,mas não aparecem 0110 ou 0010 novamente
O que isso tem de especial?

Na hora de somarmos as pontuações,os 1's aparecerão  na ordem das perguntas
respondidas corretamente!De fato,basta lembrar como se soma números
binários,como teremos sempre 1 "em cima de zero" ou "zero em cima de
zero"

Pois é,basta representar o resultado na base 2 e contar da direita para
esquerda,essa é a ordem das perguntas.Daí ele ter respondido as perguntas
2,6,7 e 10.Note que a representação na base 2 não compromete,ou melhor, ele
está por trás de tudo!Caso a pergunta 1 tivesse sido respondida corretamente
o candidato ganharia 1*2º=1 ponto,conforme expresso no enunciado.Espero ter
ajudado.

Eder

- Original Message -
From: Henrique Branco <[EMAIL PROTECTED]>
To: <[EMAIL PROTECTED]>
Sent: Monday, September 30, 2002 11:59 PM
Subject: [obm-l] Re: [obm-l] Re: [obm-l] dúvida


> Pergunta besta... Mas se 610 = (1001100010)_2, por que ele acertou as
> perguntas 2, 6, 7, 10?
> Desculpem pelo "nivel primario" da pergunta... Mas...
> Grato,
> Henrique.
>
> - Original Message -
> From: Rodrigo Villard Milet
> To: [EMAIL PROTECTED]
> Sent: Saturday, September 28, 2002 9:00 PM
> Subject: [obm-l] Re: [obm-l] dúvida
>
> Escreva 610 na base 2 : 610 = (1001100010)_2. Como sabemos que a
> representação na base 2 é única, ele acertou as perguntas 2,6,7 e 10.
> Villard
>
>
> -Mensagem original-
> De: Mário Pereira <[EMAIL PROTECTED]>
> Para: [EMAIL PROTECTED] <[EMAIL PROTECTED]>
> Data: Sábado, 28 de Setembro de 2002 11:22
> Assunto: [obm-l] dúvida
>
> Olá, se alguém puder, me dê uma dica:
>
> Em um jogo de televisão, um candidato deve responder a 10 perguntas. A
> primeira vale 1 ponto, a segunda vale 2 pontos, e assim, sucessivamente,
> dobrando sempre. O candidato responde a todas as perguntas e ganha os
pontos
> correspondentes às respostas que acertou, mesmo que erre algumas. Se o
> candidato obteve 610 pontos, quantas perguntas acertou?
>
> Obrigado,
>
> Mário.
>
>
> =
> Instruções para entrar na lista, sair da lista e usar a lista em
> http://www.mat.puc-rio.br/~nicolau/olimp/obm-l.html
> O administrador desta lista é <[EMAIL PROTECTED]>
> =

=
Instruções para entrar na lista, sair da lista e usar a lista em
http://www.mat.puc-rio.br/~nicolau/olimp/obm-l.html
O administrador desta lista é <[EMAIL PROTECTED]>
=



[obm-l] Re: [obm-l] Re: [obm-l] Re: [obm-l] dúvida

2002-10-01 Por tôpico leonardo mattos

Ola Henrique,
Se cada questao vale o dobro da anterior estando a 1ªvalendo 1ponto,as 
outras valerao sempre "potencias de 2",correto?!Colocar 610 na base é uma 
maneira de observar quantas "potencias de 2" existem nele somadas.
610=1*2^9+0*2^8+0*2^7+1*2^6+1*2^5+0*2^4+0*2^3+0*2^2+1*2^1+0*2^0
 Um abraço,Leonardo


>From: "Henrique Branco" <[EMAIL PROTECTED]>
>Reply-To: [EMAIL PROTECTED]
>To: <[EMAIL PROTECTED]>
>Subject: [obm-l] Re: [obm-l] Re: [obm-l] dúvida
>Date: Mon, 30 Sep 2002 23:59:02 -0300
>
>Pergunta besta... Mas se 610 = (1001100010)_2, por que ele acertou as
>perguntas 2, 6, 7, 10?
>Desculpem pelo "nivel primario" da pergunta... Mas...
>Grato,
>Henrique.
>
>- Original Message -
>From: Rodrigo Villard Milet
>To: [EMAIL PROTECTED]
>Sent: Saturday, September 28, 2002 9:00 PM
>Subject: [obm-l] Re: [obm-l] dúvida
>
>Escreva 610 na base 2 : 610 = (1001100010)_2. Como sabemos que a
>representação na base 2 é única, ele acertou as perguntas 2,6,7 e 10.
>Villard
>
>
>-Mensagem original-
>De: Mário Pereira <[EMAIL PROTECTED]>
>Para: [EMAIL PROTECTED] <[EMAIL PROTECTED]>
>Data: Sábado, 28 de Setembro de 2002 11:22
>Assunto: [obm-l] dúvida
>
>Olá, se alguém puder, me dê uma dica:
>
>Em um jogo de televisão, um candidato deve responder a 10 perguntas. A
>primeira vale 1 ponto, a segunda vale 2 pontos, e assim, sucessivamente,
>dobrando sempre. O candidato responde a todas as perguntas e ganha os 
>pontos
>correspondentes às respostas que acertou, mesmo que erre algumas. Se o
>candidato obteve 610 pontos, quantas perguntas acertou?
>
>Obrigado,
>
>Mário.
>
>
>=
>Instruções para entrar na lista, sair da lista e usar a lista em
>http://www.mat.puc-rio.br/~nicolau/olimp/obm-l.html
>O administrador desta lista é <[EMAIL PROTECTED]>
>=




_
Tenha você também um MSN Hotmail, o maior webmail do mundo: 
http://www.hotmail.com/br

=
Instruções para entrar na lista, sair da lista e usar a lista em
http://www.mat.puc-rio.br/~nicolau/olimp/obm-l.html
O administrador desta lista é <[EMAIL PROTECTED]>
=



[obm-l] Re: [obm-l] Re: [obm-l] Re: [obm-l] Dúvida de Combinatoria e PUTNAM

2002-08-01 Por tôpico Paulo Santa Rita

Ola Duda !
Tudo Legal ?

Concordo ! A sua interpretacao tambem e verossimil, assim como a minha. 
Todavia, em Matematica nao ha jurisprudencia, logo, interpretacao e assunto 
de outro reino, nao do espirito matematico.

Fica com Deus !
Paulo Santa Rita
5,1951,010802

>From: "Eduardo Casagrande Stabel" <[EMAIL PROTECTED]>
>Reply-To: [EMAIL PROTECTED]
>To: <[EMAIL PROTECTED]>
>Subject: [obm-l] Re: [obm-l] Re: [obm-l] Dúvida de Combinatoria e PUTNAM
>Date: Thu, 1 Aug 2002 18:45:30 -0300
>
>From: "Paulo Santa Rita" <[EMAIL PROTECTED]>
> >
> > Ola Leonardo,
> > Tudo Legal ?
> >
> > O problema esta mal formulado... Para ver isso, suponha N=3. Pelo
>enunciado
> > do problema deveriamos formar N/2=1.5 grupos ! Um absurdo, pois neste
> > contexto nao tem sentido falar em FRACAO DE GRUPO !
>
>Paulo,
>
>a correção mais natural é a seguinte:
>
>Tendo n-casais (marido e mulher) de quantas maneiras diferentes pode-se
>formar n grupos de tal forma q em que cada grupo contenha 2 pessoas,ou
>seja,não importa a ordem.Uma maneira seria
>[(H1,M1),(H2,M2),(H3,M3)...,(Hn,Mn)].
>
>Acho que esse problema nada tem a ver com permutações caóticas.
>
>Um detalhe.
>Se quisermos separar as pessoas em pares (sem que sejam casais), há
>2n!/[n!*2^n].
>Se quisermos separar as pessoas em casais, há n! maneiras.
>Está certo isso? Você concorda?
>
>Duda.
>
>
>=
>Instruções para entrar na lista, sair da lista e usar a lista em
>http://www.mat.puc-rio.br/~nicolau/olimp/obm-l.html
>O administrador desta lista é <[EMAIL PROTECTED]>
>=




_
Converse com seus amigos online, faça o download grátis do MSN Messenger: 
http://messenger.msn.com.br

=
Instruções para entrar na lista, sair da lista e usar a lista em
http://www.mat.puc-rio.br/~nicolau/olimp/obm-l.html
O administrador desta lista é <[EMAIL PROTECTED]>
=